Download as pdf or txt
Download as pdf or txt
You are on page 1of 46

TThermodynamics 647

Chapter

14
Thermodynamics

Thermodynamics is a branch of science which deals with exchange of (2) Thermodynamic variables and equation of state : A thermodynamic
heat energy between bodies and conversion of the heat energy into system can be described by specifying its pressure, volume, temperature,
mechanical energy and vice-versa. internal energy and the number of moles. These parameters are called
thermodynamic variables. The relation between the thermodynamic
Some Definitions variables (P, V, T) of the system is called equation of state.
(1) Thermodynamic system For  moles of an ideal gas, equation of state is PV = RT and for 1
mole of an it ideal gas is PV = RT
(i) It is a collection of an extremely large number of atoms or (3) Thermodynamic equilibrium : In steady state thermodynamic
molecules
variables are independent of time and the system is said to be in the state
(ii) It is confined with in certain boundaries. of thermodynamic equilibrium. For a system to be in thermodynamic
(iii) Anything outside the thermodynamic system to which energy or equilibrium, the following conditions must be fulfilled.
matter is exchanged is called its surroundings. (i) Mechanical equilibrium : There is no unbalanced force between the
system and its surroundings.
Surrounding (ii) Thermal equilibrium : There is a uniform temperature in all parts
of the system and is same as that of surrounding.
(iii) Chemical equilibrium : There is a uniform chemical composition
through out the system and the surrounding.
(4) Thermodynamic process : The process of change of state of a
Gas System system involves change of thermodynamic variables such as pressure P,
volume V and temperature T of the system. The process is known as
thermodynamic process. Some important processes are
(i) Isothermal process : Temperature remain constant
Fig. 14.1
(iv) Thermodynamic system may be of three types (ii) Adiabatic process : No transfer of heat
(a) Open system : It exchange both energy and matter with the (iii) Isobaric process : Pressure remains constant
surrounding.
(b) Closed system : It exchange only energy (not matter) with the (iv) Isochoric (isovolumic process) : Volume remains constant
surroundings. (v) Cyclic and non-cyclic process : Incyclic process Initial and final states
(c) Isolated system : It exchange neither energy nor matter with the are same while in non-cyclic process these states are different.
surrounding. (vi) Reversible and irreversible process :
648 Thermodynamics
(5) Indicator diagram : Whenever the state of a gas (P, V, T) is (iii) Change in internal energy does not depend on the path of the
changed, we say the gaseous system is undergone a thermodynamic process. process. So it is called a point function i.e. it depends only on the initial and
The graphical representation of the change in state of a gas by a final states of the system, i.e. U  U f  Ui
thermodynamic process is called indicator diagram. Indicator diagram is
plotted generally in pressure and volume of gas. (3) Work (W) : Suppose a gas is confined in a cylinder that has a
Zeroth Law of Thermodynamics movable piston at one end. If P be the pressure of the gas in the cylinder,
then force exerted by the gas on the piston of the cylinder F = PA (A = Area
If systems A and B are each in thermal equilibrium with a third system
of cross-section of piston)
C, then A and B are in thermal equilibrium with each other.
Piston
Insulating Conducting

System System System System


A B A B dx

System System F
C C Gas
Conducting Insulating

(A) (B) When the piston is pushedFig.outward


14.4
an infinitesimal distance dx, the
Fig. 14.3
(1) The zeroth law leads to the concept of temperature. All bodies in work done by the gas dW  F.dx  P( A dx )  P dV
thermal equilibrium must have a common property which has the same For a finite change in volume from V to V i f

value for all of them. This property is called the temperature.


Vf
(2) The zeroth law came to light long after the first and seconds laws
of thermodynamics had been discovered and numbered. It is so named
Total amount of work done W   Vi
P dV  P(Vf  Vi )

because it logically precedes the first and second laws of thermodynamics. (i) If we draw indicator diagram, the area bounded by PV-graph and
volume axis represents the work done
Heat, Internal Energy and Work in Thermodynamics
P
(1) Heat (Q) : It is the energy that is transferred between a system P
and its environment because of the temperature difference between them. Work = Area = P(V – V ) 2 1

(i) Heat is a path dependent quantity e.g. Heat required to change the A
temperature of a given gas at a constant pressure is different from that
required to change the temperature of same gas through same amount at
constant volume. V1 V2 V
P
(ii) For gases when heat is absorbed and temperature changes  Fig. 14.5

Q  CT V2

At constant pressure (Q)P  C P T


P Work   V1
PdV  P(V2  V1 )

At constant volume (Q)V  C V T


V1 dV V2 V
(2) Internal energy (U) : Internal energy of a system is the energy
Fig. 14.6
possessed by the system due to molecular motion and molecular
P
configuration.
The energy due to molecular motion is called internal kinetic energy U K

and that due to molecular configuration is called internal potential energy U P Work = 0
i.e. Total internal energy U  UK  UP
(i) For an ideal gas, as there is no molecular attraction U p  0
V
i.e. internal energy of an ideal gas is totally kinetic and is given by P Fig. 14.7
3 3 P2
U  UK  RT and change in internal energy U  R T
2 2 Work = Area of the shown trapezium
(ii) In case of gases whatever be the process 1
P1  (P1  P2 ) (V2  V1 )
f R  R(Tf  Ti ) 2
U   R T  C V T   T 
2 (  1)  1 V1 V2 V
Fig. 14.8
RT f  RTi (Pf V f  Pi Vi ) (ii) From W  PV  P(V f  Vi )
 
 1  1 If system expands against some external force then V f  Vi
 W = positive
TThermodynamics 649

If system contracts because of external force then V f  Vi Q = U + W


 W = negative (3) It makes no distinction between work and heat as according to it
the internal energy (and hence temperature) of a system may be increased
P B P B Negative
Positive either by adding heat to it or doing work on it or both.
work work
(4) Q and W are the path functions but U is the point function.
A
A (5) In the above equation all three quantities Q, U and W must
V V be expressed either in Joule or in calorie.
(A) Expansion (B) Compression
(6) The first law introduces the concept of internal energy.
Fig. depends
(iii) Like heat, work done is also 14.9 upon initial and final state of
the system and path adopted for the process (7) Limitation : First law of thermodynamics does not indicate the
P direction of heat transfer. It does not tell anything about the conditions,
under which heat can be transformed into work and also it does not
A
1 indicate as to why the whole of heat energy cannot be converted into
P mechanical work continuously.
A 2 B
A1
V Table 14.1 : Useful sign convention in thermodynamics
(A) Less area
1 B Quantity Sign Condition
P
V 2
+ When heat is supplied to a system
A Q
– When heat is drawn from the system
A2
B + When work done by the gas (expansion)
V W
– When work done on the gas (compression)
A<AW<W 1 2 1
(B) More area
2

Fig. 14.10
(iv) In cyclic process, work done is equal to the area of closed curve. It + With temperature rise, internal energy increases
U
is positive if the cycle is clockwise and it is negative if the cycle is – With temperature fall, internal energy decreases
anticlockwise.
P
C Isobaric Process
P2
Work = Area of triangle ABC When a thermodynamic system undergoes a physical change in such a
1 way that its pressure remains constant, then the change is known as
  (V2  V1 )  (P2  P1 ) isobaric process.
P1 A 2
B
(1) Equation of state : In this process V and T changes but P remains
V1 V2 V constant. Hence Charle’s law is obeyed in this process.
P Fig. 14.11
C V1 V2
P2 Hence if pressure remains constant V  T  
T1 T2
Work = Area of rectangle ABCD
(2) Indicator diagram : Graph 1 represent isobaric expansion, graph 2
= AB  AD
represent isobaric compression.
P1 A
B = (V – V ) (P – P )
2 1 2 1

P P
V1 V2 V
1 2
P Fig. 14.12
dP dP
P2 Slope = 0 Slope = 0
dV dV
 V V
Work = (P2  P1 ) (V2  V1 ) (A) Expansion
4 (B) Compression
P1 Fig. 14.14
(i) In isobaric expansion (Heating)
V1 V2 V
 increases so U is positive
Temperature 
Fig. 14.13
First Law of Thermodynamics (FLOT)  increases so W is positive
Volume 
(1) It is a statement of conservation of energy in thermodynamical
process.  flows into the system so Q is positive
Heat 

(2) According to it heat given to a system (Q) is equal to the sum of (ii) In isobaric compression (Cooling)
increase in its internal energy (U) and the work done (W) by the system  decreases so U is negative
Temperature 
against the surroundings.
650 Thermodynamics

 decreases so W is negative
Volume  (2) Indicator diagram : Graph 1 and 2 represent isometric increase in
pressure at volume V1 and isometric decrease in pressure at volume V2
 flows out from the system so Q is negative
Heat 
dP
respectively and slope of indicator diagram 
(3) Specific heat : Specific heat of gas during isobaric process dV
f  (i) Isometric heating
C P    1 R P
2 
(a) Pressure  increases
P 1
(4) Bulk modulus of elasticity : K   0 [As P = 0] (b) Temperature  increases
 V
V (c) Q  positive
(5) Work done in isobaric process V1 V
(d) U  positive
(A)

 
Vf Vf P
W  P dV  P dV  P[V f  Vi ] [As P = constant] (ii) Isometric cooling
Vi Vi
(a) Pressure  decreases
2
 W  P(V f  Vi )  R[T f  Ti ]  R T
(b) Temperature  decreases
(6) FLOT in isobaric process : From Q  U  W (c) Q  negative
V2 V
R (d) U  negative
 U   C V T   T and W  R T (B)
(  1) Fig. 14.17
(3) Specific heat : Specific heat of gas during isochoric process
R   
 (Q)P   T  R T     R T   CP T CV  R
f
(  1)   1  2
(7) Examples of isobaric process : All state changes occurs at constant P P
(4) Bulk modulus of elasticity : K   
temperature and pressure.  V 0
Boiling of water V
Water Vapours
(5) Work done in isochoric process
(i) Water  vapours W  PV  P[V f  Vi ] = 0 [As V = constant]
(ii) Temperature  constant (6) FLOT in isochoric process : From Q  U  W
Fig. 14.15
(iii) Volume  increases R Pf V f  Pi Vi
 W = 0  (Q)V  U   CV T   T 
(iv) A part of heat supplied is used to change volume (expansion)  1  1
against external pressure and remaining part is used to increase it's
potential energy (kinetic energy remains constant)
Isothermal Process
(v) From FLOT Q = U + W  mL = U + P(V – V ) When a thermodynamic system undergoes a physical change in such a
way that its temperature remains constant, then the change is known as
f i

Freezing of water
isothermal changes.
Water Ice
(i) Water  ice (1) Essential condition for isothermal process
(i) The walls of the container must be perfectly conducting to allow
(ii) Temperature  constant
free exchange of heat between the gas and its surrounding.
(iii) Volume  increases Fig. 14.16 (ii) The process of compression or expansion should be so slow so as
to provide time for the exchange of heat.
(iv) Heat is given by water it self. It is used to do work against
external atmospheric pressure and to decreases the internal potential Since these two conditions are not fully realised in practice, therefore,
energy. no process is perfectly isothermal.

(v) From FLOT Q = U + W  – mL = U + P(V – V ) (2) Equation of state : In this process, P and V change but T =
f i

constant i.e. change in temperature T = 0.


Isochoric or Isometric Process
Boyle’s law is obeyed i.e. PV= constant  P V = P V
When a thermodynamic process undergoes a physical change in such a 1 1 2 2

way that its volume remains constant, then the change is known as (3) Example of isothermal process : Melting of ice (at 0°C) and boiling
isochoric process. of water (at 100°C) are common example of this process.
(1) Equation of state : In this process P and T changes but V = (4) Indicator diagram : According to PV = constant, graph between P
constant. Hence Gay-Lussac’s law is obeyed in this process i.e. P  T  and V is a part of rectangular hyperbola. The graphs at different temperature
P1 P are parallel to each other are called isotherms.
 2  constant P
T1 T2
T1<T2< T3
Two isotherms never intersect
T3
T2
T1
V
Fig. 14.18
TThermodynamics 651

(ii) The system should be compressed or allowed to expand suddenly


so that there is no time for the exchange of heat between the system and its
surroundings.
Since, these two conditions are not fully realised in practice, so no
(i) Slope of isothermal curve : By differentiating PV = constant. We get process is perfectly adiabatic.
(2) Some examples of adiabatic process
P dV  V dP  0 P
(i) Sudden compression or expansion of a gas in a container with
 PdV   VdP perfectly non-conducting walls.
dP P (ii) Sudden bursting of the tube of bicycle tyre.
 Slope = tan   
dV V (iii) Propagation of sound waves in air and other gases.
 V (iv) Expansion of steam in the cylinder of steam engine.
(ii) Area between the isotherm and volume axisFig.
represents
14.19 the work
done in isothermal process. (3) FLOT in adiabatic process : From Q  U  W

If volume increases W = + (Area under curve) and if volume For adiabatic process Q  0  U   W
decreases W = – (Area under curve) If W = positive then U = negative so temperature decreases i.e.
(5) Specific heat : Specific heat of gas during isothermal change is adiabatic expansion produce cooling.

infinite. As C 
Q

Q
 [As T = 0] If W = negative then U = positive so temperature increases i.e.
m T m  0 adiabatic compression produce heating.

(6) Isothermal elasticity (E) : For this process PV = constant. (4) Equation of state : In adiabatic change ideal gases do not obeys
Boyle's law but obeys Poisson's law. According to it
dP Stress
 P dV  V dP  P    E CP
 dV / V Strain PV = constant; where  
CV
 E  P i.e. isothermal elasticity is equal to pressure
(i) For temperature and volume
At N.T.P., E = Atmospheric pressure = 1.01  10 N / m 5 2
TV = constant  T1V1 1  T2 V2 1 or T  V 1
–1

(7) Work done in isothermal process


(ii) For temperature and pressure
Vf Vf RT
W  Vi
P dV   Vi V
dV [As PV = RT] T  1   1 
= const.  T1 P1  T2 P2 or T  P
 1

or P  T  1

P  1
V  V 
W  RT loge  f   2.303 RT log10  f
 V

 Table 14.2 : Special cases of adiabatic process
 Vi   i 
1  1
P    Type of gas P T
  2 .303  RT log10  Pi V P  T  1  1
or W   RT loge  i  V
 Pf   Pf 
    Monoatomic 1 1
P P  T5 / 2 T 
(8) FLOT in isothermal process : From Q  U  W  = 5/3 V5 / 3 V2/3
Diatomic 1 1
 U  0 [As T = 0]  Q  W P P  T7 / 2 T 
 = 7/5 V7 / 5 V2/5
i.e. heat supplied in an isothermal change is used to do work against Polyatomic 1 PT 1
4
external surrounding. P T 
 = 4/3 V 4/3
V1 / 3
or if the work is done on the system than equal amount of heat energy
will be liberated by the system. (5) Indicator diagram
Adiabatic Process (i) Curve obtained on PV graph are called adiabatic curve.
When a thermodynamic system undergoes a change in such a way that (ii) Slope of adiabatic curve : From PV   constant
no exchange of heat takes place between System and surroundings, the
process is known as adiabatic process. By differentiating, we get P

In this process P, V and T changes but Q = 0. dP V   PV  1 dV  0


(1) Essential conditions for adiabatic process
dP PV  1 P
(i) There should not be any exchange of heat between the system and   
   
dV V V 
its surroundings. All walls of the container and the piston must be perfectly V
Fig. 14.20
insulating.
652 Thermodynamics

P
 Slope of adiabatic curve tan      Stop cock
V 
Gas
P Vacuum
(iii) But we also know that slope of isothermal curve tan  
V
(Slope)Adi
Hence (Slope) =   (Slope) or 1
Adi Iso

(Slope)Iso Insulated
W = 0 (Because walls are rigid)
Fig. 14.23
(6) Specific heat : Specific heat of a gas during adiabatic change is
Q = 0 (Because walls are insulated)
Q 0
zero As C   0 [As Q = 0]
m T m T U = U – U = 0 (Because Q and W are zero. Thus the final and
f i

initial energies are equal in free expansion.



(7) Adiabatic elasticity (E) : PV  constant
Cyclic and Non-cyclic Process
Differentiating both sides d PV   PV  1 dV  0
A cyclic process consists of a series of changes which return the
dP Stress
P    E  E   P system back to its initial state.
 dV / V Strain
In non-cyclic process the series of changes involved do not return the
i.e. adiabatic elasticity is  times that of pressure system back to its initial state.
E CP (1) In case of cyclic process as U f  Ui  U  U f  Ui  0
Also isothermal elasticity E  P   
E CV
i.e. change in internal energy for cyclic process is zero and also
i.e. the ratio of two elasticity of gases is equal to the ratio of two U  T  T = 0 i.e. temperature of system remains constant.
specific heats.
(2) From FLOT Q  U  W  Q  W
(8) Work done in adiabatic process
[PiVi  Pf Vf ] R(Ti  Tf ) i.e. heat supplied is equal to the work done by the system.
 
Vf Vf
K
W P dV  dV  W   (3) For cyclic process P-V graph is a closed curve and area enclosed by the
Vi Vi V (  1) (  1)
closed path represents the work done.
(As PV = K, P V = RT and P V = RT ) f f f i i i
If the cycle is clockwise work done is positive and if the cycle is
(i) W  quantity of gas (either M or ) anticlockwise work done is negative.
(ii) W  temperature difference (T – T) i f P P
1 D
(iii) W    mono   di   tri  W < W < W A B A
 1 mono dia tri

Positive work Negative work


(9) Comparison between isothermal and adiabatic indicator diagrams :
Always remember that adiabatic curves are more steeper than isothermal D C B C
curves V V
(i) Equal expansion : Graph 1 represent isothermal process and 2 (A) (B)
represent adiabatic process Fig. 14.24
P (4) Work done in non cyclic process depends upon the path chosen or
W >W isothermal adiabatic
the series of changes involved and can be calculated by the area covered
P >P between the curve and volume axis on PV diagram.
isothermal adiabatic
1
T >T P P
isothermal adiabatic
2 C B A
(Slope) < (Slope) Isothermal Adiabatic
V
Vi Vf D
B C
Fig. 14.21
(ii) Compression : Graph 1 represent adiabatic process and 2 represent
isothermal process P A
W >W V V
Adiabatic Isothermal
WABC= + Shaded area WABCD= – Shaded area
P >P 1
Adiabatic Isothermal
(A) (B)
T >T
Adiabatic Isotherm al
2 Fig. 14.25
(Slope) < (Slope) Isothermal Adiabatic
V
Quasi Static Process
Vf Vi
When we perform a process on a given system, its state is, in general,
Fig. 14.22in which no
(10) Free expansion : Free expansion is adiabatic process changed. Suppose the initial state of the system is described by the values
work is performed on or by the system. Consider two vessels placed in a P1 , V1 , T1 and the final state by P2 , V2 , T2 . If the process is performed in
system which is enclosed with thermal insulation (asbestos-covered). One such a way that at any instant during the process, the system is very nearly
vessel contains a gas and the other is evacuated. When suddenly the in thermodynamic equilibrium, the process is called quasi-static. This means,
stopcock is opened, the gas rushes into the evacuated vessel and expands we can specify the parameters P, V, T uniquely at any instant during such a
freely. process.
TThermodynamics 653

Actual processes are not quasi-static. To change the pressure of a gas, (iv) Produced by the passage of an electric current through a
we can move a piston inside the enclosure. The gas near the piston is acted resistance is irreversible.
upon by piston. The pressure of the gas may not be uniform everywhere (v) Heat transfer between bodies at different temperatures is also
while the piston is moving. However, we can move the piston very slowly to irreversible.
make the process as close to quasi-static as we wish. Thus, a quasi-static
process is an idealised process in which all changes take place infinitely (vi) Joule-Thomson effect is irreversible because on reversing the flow
slowly. of gas a similar cooling or heating effect is not observed.

Reversible and Irreversible Process Mixed Graphical Representation


(1) Reversible process : A reversible process is one which can be (1) PV-graphs
reversed in such a way that all changes occurring in the direct process are
P 4
exactly repeated in the opposite order and inverse sense and no change is 3 1  Isobaric (P-constant)
2
left in any of the bodies taking part in the process or in the surroundings.
1
For example if heat is absorbed in the direct process, the same amount of 2  Isothermal (Because P  )
heat should be given out in the reverse process, if work is done on the V
1
working substance in the direct process then the same amount of work
1
should be done by the working substance in the reverse process. The 3  Adiabatic (Because P  )
conditions for reversibility are V
O V
(i) There must be complete absence of dissipative forces such as Fig. 14.26 4  Isochoric (V-constant)
friction, viscosity, electric resistance etc.
(2) PT-graphs
(ii) The direct and reverse processes must take place infinitely slowly.
P 2 3
(iii) The temperature of the system must not differ appreciably from 4 1  Isobaric (P-constant)
its surroundings.
1 2  Isothermal (T-constant)
Some examples of reversible process are

(a) All isothermal and adiabatic changes are reversible if they are
3  Adiabatic (Because P  T  1 )
performed very slowly.
T
(b) When a certain amount of heat is absorbed by ice, it melts. If the O 4  Isochoric (In isochoric P  T)
Fig. 14.27
same amount of heat is removed from it, the water formed in the direct
process will be converted into ice. (3) VT-graphs
(c) An extremely slow extension or contraction of a spring without V 3
setting up oscillations. 2 1  Isochoric (V-constant)
4
(d) When a perfectly elastic ball falls from some height on a perfectly 1

elastic horizontal plane, the ball rises to the initial height. 1 2  Adiabatic (Because V  T 1  )
(e) If the resistance of a thermocouple is negligible there will be no
3  Isothermal (T-constant)
heat produced due to Joule’s heating effect. In such a case heating or
cooling is reversible. At a junction where a cooling effect is produced due to T
O 4  Isobaric (In isobaric V  T )
Peltier effect when current flows in one direction and equal heating effect is Fig. 14.28
produced when the current is reversed.
(f) Very slow evaporation or condensation.
Heat Engine
It should be remembered that the conditions mentioned for a Heat engine is a device which converts heat into work continuously
through a cyclic process.
reversible process can never be realised in practice. Hence, a reversible
process is only an ideal concept. In actual process, there is always loss of The essential parts of a heat engine are
heat due to friction, conduction, radiation etc. (1) Source : It is a reservoir of heat at high temperature and infinite
(2) Irreversible process : Any process which is not reversible exactly is an thermal capacity. Any amount of heat can be extracted from it.
irreversible process. All natural processes such as conduction, radiation, (2) Working substance : Steam, petrol etc.
radioactive decay etc. are irreversible. All practical processes such as free
(3) Sink : It is a reservoir of heat at low temperature and infinite
expansion, Joule-Thomson expansion, electrical heating of a wire are also
thermal capacity. Any amount of heat can be given to the sink.
irreversible. Some examples of irreversible processes are given below
(i) When a steel ball is allowed to fall on an inelastic lead sheet, its
kinetic energy changes into heat energy by friction. The heat energy raises Source (T1)
the temperature of lead sheet. No reverse transformation of heat energy
occurs. Q1
Heat W = Q 1 – Q2
(ii) The sudden and fast stretching of a spring may produce vibrations Engine
in it. Now a part of the energy is dissipated. This is the case of irreversible Q2
process.
(iii) Sudden expansion or contraction and rapid evaporation or Sink (T2)
condensation are examples of irreversible processes. The working substance absorbs heat Q from the source, does an 1

amount of work W, returns theFig. 14.29


remaining amount of heat to the sink and
654 Thermodynamics
comes back to its original state and there occurs no change in its internal Q1  Q 2 T1  T2 Q2 T2
energy.   or 
Q2 T2 Q1  Q 2 T1  T2
By repeating the same cycle over and over again, work is continuously
T2
obtained. So coefficient of performance  
T1  T2
The performance of heat engine is expressed by means of “efficiency”
 which is defined as the ratio of useful work obtained from the engine to where T = temperature of surrounding, T = temperature of cold body.
1 2

the heat supplied to it. It is clear that  = 0 when T = 0


2

i.e. the coefficient of performance will be zero if the cold body is at the
Work done W
  temperature equal to absolute zero.
Heat input Q1
(2) Relation between coefficient of performance and efficiency of
For cyclic process U = 0 hence from FLOT Q = W refrigerator

Q1  Q 2 Q2 Q2 / Q1
So W  Q1  Q2   
Q
1 2 We know    ….. (i)
Q1 Q1 Q1  Q 2 1  Q2 / Q1

A perfect heat engine is one which converts all heat into work i.e. Q2 Q
But the efficiency   1  or 2  1   …..(ii)
W  Q1 so that Q 2  0 and hence   1 . Q1 Q1

But practically efficiency of an engine is always less than 1. 1 


From (i) and (ii) we get,  

Refrigerator or Heat Pump
Second Law of Thermodynamics
A refrigerator or heat pump is basically a heat engine run in reverse
direction. First law of thermodynamics merely explains the equivalence of work
and heat. It does not explain why heat flows from bodies at higher
It essentially consists of three parts
temperatures to those at lower temperatures. It cannot tell us why the
(1) Source : At higher temperature T . 1 converse is possible. It cannot explain why the efficiency of a heat engine is
(2) Working substance : It is called refrigerant liquid ammonia and always less than unity. It is also unable to explain why cool water on stirring
freon works as a working substance. gets hotter whereas there is no such effect on stirring warm water in a
beaker. Second law of thermodynamics provides answers to these questions.
(3) Sink : At lower temperature T . 2
Statement of this law is as follows
Source
(T1) (1) Clausius statement : It is impossible for a self acting machine to
(Atmosphere)
transfer heat from a colder body to a hotter one without the aid of an
Q1
external agency.
Heat W = Q 1 – Q2
Engine From Clausius statement it is clear that heat cannot flow from a
Q2
body at low temperature to one at higher temperature unless work is done
by an external agency. This statement is in fair agreement with our
Sink
(Contents of refrigerator) (T2)
experiences in different branches of physics. For example, electrical current
cannot flow from a conductor at lower electrostatic potential to that at
Fig. 14.30 higher potential unless an external work is done. Similarly, a body at a lower
The working substance takes heat Q from a sink (contents of
gravitational potential level cannot move up to higher level without work
2

refrigerator) at lower temperature, has a net amount of work done W on it


by an external agent (usually compressor of refrigerator) and gives out a done by an external agency.
larger amount of heat Q to a hot body at temperature T (usually
1 1 (2) Kelvin’s statement : It is impossible for a body or system to
atmosphere). Thus, it transfers heat from a cold to a hot body at the perform continuous work by cooling it to a temperature lower than the
expense of mechanical energy supplied to it by an external agent. The cold
temperature of the coldest one of its surroundings. A Carnot engine cannot
body is thus cooled more and more.
work if the source and sink are at the same temperature because work done
The performance of a refrigerator is expressed by means of “coefficient by the engine will result into cooling the source and heating the
of performance”  which is defined as the ratio of the heat extracted from surroundings more and more.
the cold body to the work needed to transfer it to the hot body.
(3) Kelvin-Planck’s statement : It is impossible to design an engine
Heat extracted Q2 Q2 that extracts heat and fully utilises into work without producing any other
i.e.    
Work done W Q1  Q2 effect.
A perfect refrigerator is one which transfers heat from cold to hot From this statement it is clear that any amount of heat can never be
body without doing work converted completely into work. It is essential for an engine to return some
i.e. W = 0 so that Q1  Q 2 and hence    amount of heat to the sink. An engine essentially requires a source as well
as sink. The efficiency of an engine is always less than unity because heat
Q1 T cannot be fully converted into work.
(1) Carnot refrigerator : For Carnot refrigerator  1
Q2 T2
Carnot Engine
TThermodynamics 655

(1) Carnot designed a theoretical engine which is free from all the V4 V4
defects of a practical engine. This engine cannot be realised in actual W3  Q2    V3
P dV   RT2 loge
V3
practice, however, this can be taken as a standard against which the
performance of an actual engine can be judged. V3
 RT2 loge  Area CDFH
V4
(iv) Fourth stroke (adiabatic compression) (curve DA) : Finally the
cylinder is again placed on non-conducting stand and the compression is
continued so that gas returns to its initial stage.
Ideal gas
V1 R R
W4    V4
P dV  
 1
(T2  T1 ) 
 1
(T1  T2 )  Area ADFE

Source Insulating Sink (3) Efficiency of Carnot cycle : The efficiency of engine is defined as
T1 K stand T2 K the ratio of work done to the heat supplied i.e.
Work done W
 
It consists of the following parts Heat input Q1
Fig. 14.31
(i) A cylinder with perfectly non-conducting walls and a perfectly Net work done during the complete cycle
conducting base containing a perfect gas as working substance and fitted
with a non-conducting frictionless piston W  W1  W2  (W3 )  (W4 )  W1  W3  Area ABCD
(ii) A source of infinite thermal capacity maintained at constant higher [As W2  W4 ]
temperature T 1.

(iii) A sink of infinite thermal capacity maintained at constant lower W W1  W3 Q1  Q 2 W Q


    1 3 1 2
temperature T . 2
Q1 W1 Q1 W1 Q1
(iv) A perfectly non-conducting stand for the cylinder. RT2 loge (V3 / V4 )
or   1 
(2) Carnot cycle : As the engine works, the working substance of the RT1 loge (V2 / V1 )
engine undergoes a cycle known as Carnot cycle. The Carnot cycle consists
Since points B and C lie on same adiabatic curve
of the following four strokes
 1
P T1  V3 
 T1 V2 1  T2 V3 1 or   …..(i)
A(P1, V1) Q1 T2  V2 

T1 Also point D and A lie on the same adiabatic curve


 1
W = Q1 – Q2 T1  V4 
 T1 V1 1  T2 V4 1 or   …..(ii)
T2  V1 

(P4, V4)D V3 V4 V V V  V 
T2 From (i) and (ii),  or 3  2  loge  3   loge  2 
C(P3, V3)
Q2 V2 V1 V4 V1  4
V  V1 
V
E F G H T2
So efficiency of Carnot engine   1 
(i) First stroke (IsothermalFig.expansion)
14.32 (curve AB) : T1
The cylinder containing ideal gas as working substance allowed to (i) Efficiency of a heat engine depends only on temperatures of source and
expand slowly at this constant temperature T . 1
sink and is independent of all other factors.
Work done = Heat absorbed by the system (ii) All reversible heat engines working between same temperatures are
equally efficient and no heat engine can be more efficient than Carnot
V2 V 
W1  Q1   V1
P dV  RT1 loge  2
 V1
  Area ABGE


engine (as it is ideal).
(iii) As on Kelvin scale, temperature can never be negative (as 0 K is
(ii) Second stroke (Adiabatic expansion) (curve BC) : defined as the lowest possible temperature) and T and T are finite, l 2

The cylinder is then placed on the non conducting stand and the gas is efficiency of a heat engine is always lesser than unity, i.e., whole of heat can
allowed to expand adiabatically till the temperature falls from T to T . 1 2
never be converted into work which is in accordance with second law.
V3 R (4) Carnot theorem : The efficiency of Carnot’s heat engine depends
W2   V2
P dV =
(  1)
[T1  T2 ]  Area BCHG only on the temperature of source (T ) and temperature of sink (T ), i.e.,
1 2

T
(iii) Third stroke (Isothermal compression) (curve CD) :  1 2 .
T1
The cylinder is placed on the sink and the gas is compressed at
constant temperature T . Carnot stated that no heat engine working between two given
2

temperatures of source and sink can be more efficient than a perfectly


Work done = Heat released by the system reversible engine (Carnot engine) working between the same two
656 Thermodynamics
temperatures. Carnot's reversible engine working between two given
temperatures is considered to be the most efficient engine.
Table 14.3 : Difference Between Petrol Engine and Diesel Engine  When a thermos bottle is vigorously shaken :
Petrol engine Diesel engine Heat transferred to the coffee  Q = 0
Working substance is a mixture of Working substance in this engine is a [As thermos flask is insulated from the surrounding]
petrol vapour and air. mixture of diesel vapour and air.
Work is done on the coffee against viscous force
Efficiency is smaller (~47%). Efficiency is larger (~55%).
W = (–)
It works with a spark plug. It works with an oil plug.
Internal energy of the coffee increases U = (+)
It is associated with the risk of No risk of explosion, because only air
explosion, because petrol vapour and is compressed. Hence compression and temperature of the coffee also increases T = (+)
air is compressed. So, low compression ratio is kept large.
ratio is kept.
 Work done without the volume limits
Vf
Petrol vapour and air is created with
spark plug.
Spray of diesel is obtained through
the jet.
W  Vi
P dV

From this equation it seems as if work done can be calculated only


Entropy
when P-V equation is known and limits Vi and V f are known to us.
Entropy is a measure of disorder of molecular motion of a system. But it is not so. We can calculate work done if we know the limits of
Greater is the disorder, greater is the entropy. temperature.
The change in entropy i.e. 
For  moles of an ideal gas if P  with temperature limits T 0

Heat absorbed by system dQ T


dS  or dS  and 2T .
Absolutetemperatu re T 0

The relation is called the mathematical form of Second Law of RT RT 2 2 RT
From PV = RT  V    dV  dT
Thermodynamics. P  
(1) For solids and liquids Vf 2 T0     2 RT 
(i) When heat given to a substance changes its state at constant  W  Vi
P dV   T0
 
 T   
 dT  2 RT0
dQ mL
temperature, then change in entropy dS  
T T  Work done with spring : If mass less piston is attached to a spring of
where positive sign refers to heat absorption and negative sign to heat force constant K and a mass m is placed over the piston. If the external
evolution. pressure is P0 and due to expansion of gas the piston moves up through
(ii) When heat given to a substance raises its temperature from T to 1
a distance x then
T , then change in entropy
2 Total work done by the gas
dQ T2 dT T  W  W1  W2  W3
dS   T
  T1
mc
T
 mc loge  2
 T1


 where W = Work done against
M
1

M x
T  external pressure (P 0 )
 S  2 .303 mc log10  2 .

 T1  W = Work done against spring
2

P
(2) For a perfect gas : Perfect gas equation for n moles is PV = nRT force (Kx )
dQ CV dT  P dV W = Work done against gravitational force (mg)
S   T
  T
[As dQ = dU + dW] 3

1
 W  P0 V  Kx 2  mgx
 RT 2
CV dT  dV
 S   T
V
 The efficiency of an actual engine is much lesser than that of an
ideal engine. Actually the practical efficiency of a steam engine is about
T2 dT V2 dV
 CV  T1 T
 R  V1 V
[As PV =  RT] (8-15)% while that of a petrol engine is 40%. The efficiency of a diesel
engine is maximum and is about (50-55)%.
T  V 
 S  CV loge  2   R loge  2 

 When P and V bear the relation PV = constant, where x  1 or 
x

 1
T  V1  the process is called a polytropic one. In this process the molar heat
T  P  R R R
In terms of T and P, S  C P loge  2   R loge  2  capacity is, C  CV   
 P  1  x  1 1  x
 T1   1 
P  V   Enthalpy : Four quantities called “thermodynamic potentials” are
and in terms of P and V S  CV loge  2   C P loge  2 
 1
P  V1  useful in the chemical thermodynamics of reactions and non-cyclic
processes. They are internal energy, the enthalpy, the Helmoltz free
energy and the Gibbs free energy. Enthalpy is defined by
TThermodynamics 657

H = U + PV 5. The temperature of an ideal gas is kept constant as it expands. The


gas does external work. During this process, the internal energy of
where P and V are the pressure and volume, and U is internal energy, the gas [MP PMT 1990]
Enthalpy is somewhat parallel to the first law of thermodynamics for a
(a) Decreases
constant pressure system Q = U + PV since in this case Q = H.
(b) Increases
 Confusion about FLOT (c) Remains constant
It is typical for chemistry texts to write the first law as
(d) Depends on the molecular motion
U = Q + W 6. The first law of thermodynamics is concerned with the conservation
It is the same law, of course the thermodynamic expression of the of [MP PMT 1987; CBSE PMT 1990, 92;
conservation of energy principle. It is just that W is defined as the work AFMC 1997; CPMT 1999; BHU 1999;
done on the system instead of work done by the system. In the context DCE 2000; BCECE 2003]
of physics, the common scenario is one of adding heat to a volume of (a) Momentum (b) Energy
gas and using the expansion of that gas to do work, as in the pushing
down of a piston in an internal combustion engine. In the context of (c) Mass (d) Temperature
chemical reactions and process, it may be more common to deal with 7. A thermodynamic system goes from states (i) P1 , V to 2P1 , V (ii)
situations where work is done on the system rather than by it.
P, V to P, 2V. Then work done in the two cases is
 Possibilities [MP PMT 1990]
P
(a) Zero, Zero (b) Zero, PV1

a (c) PV1 , Zero (d) PV1 , P1V1

b 8. If the amount of heat given to a system be 35 joules and the amount


of work done by the system be 15 joules, then the change in the
V internal energy of the system is
If a  Isothermal then b  Must be adiabatic
[MP PMT 1989]
But If b  adiabatic then it is not compulsory that a must be
isothermal, it may be adiabatic also. (a) 50 joules (b) 20 joules
(c) 30 joules (d) 50 joules
9. A system is given 300 calories of heat and it does 600 joules of
work. How much does the internal energy of the system change in
this process
(J = 4.18 joules/cal) [MP PET 1991]
(a) 654 Joule (b) 156.5 Joule
First Law of Thermodynamics (Q = U + W) (c) – 300 Joule (d) – 528.2 Joule
1. First law of thermnodynamics is given by [CPMT 1977, 91] 10. Work done on or by a gas, in general depends upon the
(a) Initial state only
(a) dQ  dU  PdV (b) dQ  dU  PdV
(b) Final state only
(c) dQ  (dU  dV ) P (d) dQ  PdU  dV
(c) Both initial and final states only
2. The internal energy of an ideal gas depends upon (d) Initial state, final state and the path
[RPMT 1997; MP PMT 1999; CPMT 2003]
11. If R = universal gas constant, the amount of heat needed to raise the
(a) Specific volume (b) Pressure temperature of 2 mole of an ideal monoatomic gas from 273K to
(c) Temperature (d) Density 373K when no work is done [MP PET 1990]

3. In changing the state of thermodynamics from A to B state, the heat (a) 100 R (b) 150 R
required is Q and the work done by the system is W. The change in (c) 300 R (d) 500 R
its internal energy is
[MP PMT 1986; AMU (Med.) 2001]
12. Find the change in internal energy of the system when a system
absorbs 2 kilocalorie of heat and at the same time does 500 joule of
(a) Q + W (b) Q – W work [EAMCET 1984]
Q W (a) 7900 J (b) 8200 J
(c) Q (d)
2 (c) 5600 J (d) 6400 J
4. Heat given to a system is 35 joules and work done by the system is
15 joules. The change in the internal energy of the system will be [MP PET/PMT 1988]
(a) – 50 J (b) 20 J
(c) 30 J (d) 50 J
Thermodynamics 659
[MH CET 1999]
13. A system performs work W when an amount of heat is Q
added to the system, the corresponding change in the internal (a) Remains constant (b) Becomes zero
energy is U . A unique function of the initial and final states (c) Increases (d) Decreases
(irrespective of the mode of change) is 22. If 150 J of heat is added to a system and the work done by the
[CPMT 1981; J & KCET 2004]
system is 110 J, then change in internal energy will be
[AMU (Engg.) 1999; BHU 2000]
(a) Q (b) W
(a) 260 J (b) 150 J
(c) U and Q (d) U (c) 110 J (d) 40 J

14. A container of volume 1m 3 is divided into two equal compartments 23. If Q and W represent the heat supplied to the system and the
by a partition. One of these compartments contains an ideal gas at work done on the system respectively, then the first law of
300 K. The other compartment is vacuum. The whole system is thermodynamics can be written as [Roorkee 2000]
thermally isolated from its surroundings. The partition is removed (a) Q  U  W (b) Q  U  W
and the gas expands to occupy the whole volume of the container.
Its temperature now would be [Manipal MEE 1995] (c) Q  W  U (d) Q  W  U

(a) 300 K (b) 239 K where U is the internal energy


(c) 200 K (d) 100 K 24. For free expansion of the gas which of the following is true
[AMU (Med.) 2000]
15. 110 J of heat is added to a gaseous system, whose internal energy
change is 40 J, then the amount of external work done is [CBSE PMT 1993; DPMT (a) Q
1996, 03; AFMC 0 and  Eint  0
W 1999;
JIPMER 2000; MH CET 2000; Pb. PMT 2003]
(b) Q  0, W  0 and Eint  W
(a) 150 J (b) 70 J
(c) W  0, Q  0, and  Eint  Q
(c) 110 J (d) 40 J
(d) W  0, Q  0 and Eint  0
16. Which of the following is not thermodynamical function
[CBSE PMT 1993; CPMT 2001; DCE 1996; 2001]
25. Which of the following can not determine the state of a
thermodynamic system [AFMC 2001]
(a) Enthalpy (b) Work done
(a) Pressure and volume
(c) Gibb's energy (d) Internal energy (b) Volume and temperature
17. When the amount of work done is 333 cal and change in internal (c) Temperature and pressure
energy is 167 cal, then the heat supplied is
(d) Any one of pressure, volume or temperature
[AFMC 1998]
26. Which of the following is not a thermodynamics co-ordinate
(a) 166 cal (b) 333 cal [AIIMS 2001]
(c) 500 cal (d) 400 cal (a) P (b) T
18. First law thermodynamics states that [KCET 1999] (c) V (d) R
(a) System can do work 27. In a given process for an ideal gas, dW  0 and dQ  0. Then for
(b) System has temperature the gas [IIT-JEE (Screening) 2001]
(a) The temperature will decrease
(c) System has pressure
(b) The volume will increase
(d) Heat is a form of energy
(c) The pressure will remain constant
19. A thermo-dynamical system is changed from state (P1 , V1 ) to (d) The temperature will increase
(P2 , V2 ) by two different process. The quantity which will remain 28. The specific heat of hydrogen gas at constant pressure is
same will be [RPET 1999] CP  3.4  10 3 cal / kg o C and at constant volume is
(a) Q (b)  W CV  2.4  10 cal / kg C. If one kilogram hydrogen gas is heated
3 o

(c) Q  W (d) Q  W from 10 o C to 20 o C at constant pressure, the external work


done on the gas to maintain it at constant pressure is [
20. In thermodynamic process, 200 Joules of heat is given to a gas and
4
100 Joules of work is also done on it. The change in internal energy (a) 10 5 cal (b) 10 cal
of the gas is [AMU (Engg.) 1999]
(c) 10 3 cal (d) 5  10 3 cal
(a) 100 J (b) 300 J
29. Which of the following parameters does not characterize the
(c) 419 J (d) 24 J thermodynamic state of matter [CPMT 2001; AIEEE 2003]
21. A perfect gas contained in a cylinder is kept in vacuum. If the (a) Volume (b) Temperature
cylinder suddenly bursts, then the temperature of the gas
660 Thermodynamics
(c) Pressure (d) Work energy when temperature of 2 moles of this gas is increased from
30. In a thermodynamic system working substance is ideal gas, its 340 K to 342 K [RPET 1997]
internal energy is in the form of [MP PMT 2003] (a) 27.80 cal (b) 19.84 cal
(a) Kinetic energy only (c) 13.90 cal (d) 9.92 cal
(b) Kinetic and potential energy 40. Temperature is a measurement of coldness or hotness of an object.
(c) Potential energy This definition is based on [RPET 2003]
(d) None of these (a) Zeroth law of thermodynamics
31. Which of the following statements is correct for any thermodynamic (b) First law of thermodynamics
system [AIEEE 2004] (c) Second law of thermodynamics
(a) The internal energy changes in all processes (d) Newton's law of cooling
(b) Internal energy and entropy are state functions 41. When heat energy of 1500 Joules, is supplied to a gas at constant
(c) The change in entropy can never be zero pressure 2.1  10 5 N /m 2 , there was an increase in its volume
(d) The work done in an adiabatic process is always zero equal to 2.5  10 3 m 3 . The increase in internal energy of the gas
32. A system is provided with 200 cal of heat and the work done by the in Joules is [EAMCET (Engg.) 1999]
system on the surrounding is 40 J. Then its internal energy (a) 450 [Orissa PMT 2004] (b) 525
(a) Increases by 600 J (b) Decreases by 800 J (c) 975 (d) 2025
(c) Increases by 800 J (d) Decreases by 50 J 42. If heat given to a system is 6 kcal and work done is 6 kJ. Then
33. In a thermodynamic process, pressure of a fixed mass of a gas is change in internal energy is [BHU Med. 2000]
changed in such a manner that the gas molecules gives out 20 J of (a) 19.1 kJ (b) 12.5 kJ
heat and 10 J of work is done on the gas. If the initial internal energy
of the gas was 40 J, then the final internal energy will be kJ
(c) 25[DPMT 2004] (d) Zero
(a) 30 J (b) 20 J 43. In a thermodynamics process, pressure of a fixed mass of a gas is
changed in such a manner that the gas releases 20 J of heat and 8J
(c) 60 J (d) 40 J
of work is done on the gas. If the initial internal energy of the gas
34. Heat is not being exchanged in a body. If its internal energy is was 30J. The final internal energy will be
increased, then [RPMT 2002]
(a) 18J (b) 9J
(a) Its temperature will increase
(c) 4.5J (d) 36J
(b) Its temperature will decrease
(c) Its temperature will remain constant 44. A monoatomic gas of n-moles is heated from temperature T to T 1 2

under two different conditions (i) at constant volume and (ii) at


(d) None of these constant pressure. The change in internal energy of the gas is
35. Out of the following which quantity does not depend on path [RPET
(a) More for2002]
(i)
(a) Temperature (b) Energy (b) More for (ii)
(c) Work (d) None of these (c) Same in both cases
36. First law of thermodynamics is a special case of (d) Independent of number of moles
[CPMT 1985; RPET 2000; DCE 2000;
CBSE PMT 2000; AIEEE 2002; AFMC 2002] 45. The state of a thermodynamic system is represented by
(a) Newton's law [MH CET 2004]
(b) Law of conservation of energy (a) Pressure only
(c) Charle's law (b) Volume only
(d) Law of heat exchange (c) Pressure, volume and temperature
37. One mole of an ideal monoatomic gas is heated at a constant (d) Number of moles
pressure of one atmosphere from 0 o C to 100 o C . Then the 46. A perfect gas goes from state A to another state B by absorbing
change in the internal energy is [Pb. PMT 2001] 8  10 5 J of heat and doing 6.5  10 5 J of external work. It is
now transferred between the same two states in another process in
(a) 6.56 joules (b) 8.32  10 2 joules
which it absorbs 10 5 J of heat. Then in the second process
(c) 12.48  10 joules
2
(d) 20.80 joules
(a) Work done on the gas is 0.5  10 5 J
38. If the ratio of specific heat of a gas at constant pressure to that at
constant volume is  , the change in internal energy of a mass of (b) Work done by gas is 0.5  10 5 J
gas, when the volume changes from V to 2V constant pressure p, is [CBSE PMT 1998]
(c) Work done on gas is 10 5 J
(a) R /(  1) (b) pV
(d) Work done by gas is 10 5 J
(c) pV /(  1) (d) pV /(  1) 47. If a system undergoes contraction of volume then the work done by
39. If CV  4.96cal / mole K, then increase in internal the system will be [BHU 1999]
(a) Zero (b) Negligible
Thermodynamics 661
(c) Negative (d) Positive 8. One mole of O2 gas having a volume equal to 22.4 litres at 0 o C
48. Which of the following is incorrect regarding the first law of and 1 atmospheric pressure in compressed isothermally so that its
thermodynamics [AIEEE 2005] volume reduces to 11.2 litres. The work done in this process is [
(a) It introduces the concept of the internal energy (a) 1672.5 J (b) 1728 J
(b) It introduces the concept of the entropy
(c) 1728 J (d) 1572.5 J
(c) It is not applicable to any cyclic process
9. If a gas is heated at constant pressure, its isothermal compressibility
(d) None of the above
(a) Remains constant
Isothermal Process (b) Increases linearly with temperature
(c) Decreases linearly with temperature
1. For an ideal gas, in an isothermal process [BHU 1998] (d) Decreases inversely with temperature
(a) Heat content remains constant 10. Work done per mol in an isothermal change is
(b) Heat content and temperature remain constant [RPMT 2004; BCECE 2005]

(c) Temperature remains constant V2 V1


(a) RT log10 (b) RT log10
(d) None of the above V1 V2

2. Can two isothermal curves cut each other V2 V1


(c) RT loge (d) RT loge
(a) Never V1 V2
(b) Yes 11. The isothermal Bulk modulus of an ideal gas at pressure P is
(c) They will cut when temperature is 0°C [CPMT 1974, 81; UPSEAT 1998; IIT 1998]
(d) Yes, when the pressure is critical pressure (a) P (b)  P
3. In an isothermal expansion [KCET 2000; AFMC 2001]
(c) P / 2 (d) P / 
(a) Internal energy of the gas increases 12. In isothermal expansion, the pressure is determined by
(b) Internal energy of the gas decreases [AFMC 1995]
(c) Internal energy remains unchanged (a) Temperature only
(d) Average kinetic energy of gas molecule decreases (b) Compressibility only
4. In an isothermal reversible expansion, if the volume of 96 gm of (c) Both temperature and compressibility
oxygen at 27°C is increased from 70 litres to 140 litres, then the
(d) None of these
work done by the gas will be
13. The isothermal bulk modulus of a perfect gas at normal pressure is
(a) 300 R log10 2 (b) 81 R loge 2
(a) 1.013  10 5 N / m 2 (b) 1.013  10 6 N / m 2
(c) 900 R log10 2 (d) 2.3  900 R log10 2
(c) 1., 013  10 11 N / m 2 (d) 1.013  1011 N / m 2
5. A vessel containing 5 litres of a gas at 0.8 m pressure is connected
to an evacuated vessel of volume 3 litres. The resultant pressure 14. In an isothermal change, an ideal gas obeys
inside will be (assuming whole system to be isolated) [MP PMT 1993] [EAMCET 1994; CPMT 1999]
(a) 4/3 m (b) 0.5 m (a) Boyle's law (b) Charle's law
(c) 2.0 m (d) 3/4 m (c) Gaylussac law (d) None of the above
P 15. In isothermic process, which statement is wrong
6. For an isothermal expansion of a perfect gas, the value of is [RPMT 1997]
P
equal [CPMT 1980] (a) Temperature is constant
V V (b) Internal energy is constant
(a) 1/2 (b) 
V V (c) No exchange of energy
V V (d) (a) and (b) are correct
(c)  (d)   2
V V 16. An ideal gas A and a real gas B have their volumes increased from V
PV to 2 V under isothermal conditions. The increase in internal energy
7. The gas law  constant is true for [CBSE PMT 1993; JIPMER 2001, 02]
T
[MNR 1974; MP PMT 1984; BHU 1995, 98, 2000] (a) Will be same in both A and B
(a) Isothermal changes only (b) Will be zero in both the gases
(c) Of B will be more than that of A
(b) Adiabatic changes only
(d) Of A will be more than that of B
(c) Both isothermal and adiabatic changes
17. The specific heat of a gas in an isothermal process is
(d) Neither isothermal nor adiabatic changes
[AFMC 1998]
(a) Infinite (b) Zero
(c) Negative (d) Remains constant
662 Thermodynamics
18. A thermally insulated container is divided into two parts by a screen. 27. 540 calories of heat convert 1 cubic centimeter of water at 100 o C
In one part the pressure and temperature are P and T for an ideal
gas filled. In the second part it is vacuum. If now a small hole is into 1671 cubic centimeter of steam at 100 o C at a pressure of one
created in the screen, then the temperature of the gas will atmosphere.
[RPET Then
1999] the work done against the atmospheric pressure
is nearly
(a) Decrease (b) Increase
(a) 540 cal (b) 40 cal
(c) Remain same (d) None of the above
(c) Zero cal (d) 500 cal
19. A container that suits the occurrence of an isothermal process
should be made of [Pb. PMT 2000] 28. One mole of an ideal gas expands at a constant temperature of 300
K from an initial volume of 10 litres to a final volume of 20 litres.
(a) Copper (b) Glass The work done in expanding the gas is
(c) Wood (d) Cloth (R = 8.31 J/mole-K) [MP PMT 1995; UPSEAT 2000]
20. In an isothermal process the volume of an ideal gas is halved. One (a) 750 joules (b) 1728 joules
can say that [MP PMT 2004]
(c) 1500 joules (d) 3456 joules
(a) Internal energy of the system decreases
29. A cylinder fitted with a piston contains 0.2 moles of air at
(b) Work done by the gas is positive temperature 27°C. The piston is pushed so slowly that the air within
(c) Work done by the gas is negative the cylinder remains in thermal equilibrium with the surroundings.
Find the approximate work done by the system if the final volume is
(d) Internal energy of the system increases
twice the initial volume
21. A thermodynamic process in which temperature T of the system
[BHU (Med.) 2000]
remains constant though other variable P and V may change, is
called [Pb. PMT 2004] (a) 543 J (b) 345 J
(a) Isochoric process (b) Isothermal process (c) 453 J (d) 600 J
(c) Isobaric process (d) None of these 30. The volume of an ideal gas is 1 litre and its pressure is equal to
72cm of mercury column. The volume of gas is made 900 cm by 3

22. If an ideal gas is compressed isothermally then [RPMT 2003] compressing it isothermally. The stress of the gas will be
(a) No work is done against gas (a) 8 cm (mercury) (b) 7 cm (mercury)
(b) Heat is relased by the gas (c) 6 cm (mercury) (d) 4 cm (mercury)
(c) The internal energy of gas will increase 31. During an isothermal expansion of an ideal gas
[UPSEAT 2005]
(d) Pressure does not change
(a) Its internal energy decreases
23. When an ideal gas in a cylinder was compressed isothermally by a (b) Its internal energy does not change
piston, the work done on the gas was found to be (c) The work done by the gas is equal to the quantity of heat
1.5  10 4 joules . During this process about absorbed by it
[MP PMT 1987] (d) Both (b) and (c) are correct
(a) 3.6  10 cal of heat flowed out from the gas
3
Adiabatic Process
(b) 3.6  10 3 cal of heat flowed into the gas 1. If a cylinder containing a gas at high pressure explodes, the gas
(c) 1.5  10 4 cal of heat flowed into the gas undergoes [MP PET/PMT 1988]

(d) 1.5  10 4 cal of heat flowed out from the gas (a) Reversible adiabatic change and fall of temperature
24. When heat is given to a gas in an isothermal change, the result will (b) Reversible adiabatic change and rise of temperature
be [MP PET 1995; RPMT 1997] (c) Irreversible adiabatic change and fall of temperature
(a) External work done (d) Irreversible adiabatic change and rise of temperature
(b) Rise in temperature 2. The work done in an adiabatic change in a gas depends only on [CPMT 1971; MP
(c) Increase in internal energy (a) Change is pressure (b) Change is volume
(d) External work done and also rise in temp. (c) Change in temperature (d) None of the above
25. When 1 gm of water at 0 o C and 1  10 5 N / m 2 pressure is 3. In adiabatic expansion [DPMT 1999]

converted into ice of volume 1.091 cm 2 , the external work done (a) U  0 (b) U  negative
will be (c) U  positive (d) W  zero
(a) 0.0091 joule (b) 0.0182 joule 4. The pressure in the tyre of a car is four times the atmospheric
pressure at 300 K. If this tyre suddenly bursts, its new temperature
(c) – 0.0091 joule (d) – 0.0182 joule will be (  1.4 )
26. The latent heat of vaporisation of water is 2240 J/gm. If the work
[RPMT 1996; MP PMT 1990]
done in the process of expansion of 1 g is 168 J, then increase in
internal energy is [Pb. PET 1998; CPMT 2000] 0 .4 / 1 .4
1
(a) 300 (4 )1.4 / 0.4 (b) 300  
(a) 2408 J (b) 2240 J 4
(c) 2072 J (d) 1904 J
(c) 300 (2)0.4 / 1.4 (d) 300 (4 )0.4 / 1.4
Thermodynamics 663
5. A gas at NTP is suddenly compressed to one-fourth of its original 1
3 (d) Adiabatic curve slope =  isothermal curve slope
volume. If  is supposed to be , then the final pressure is [BHU 1995] 2
2 12. Pressure-temperature relationship for an ideal gas undergoing
3 adiabatic change is (  C p / Cv )
(a) 4 atmosphere (b) atmosphere
2
[CPMT 1992; MP PMT 1986, 87, 94, 97; Pb. PET 1998;
1
(c) 8 atmosphere (d) atmosphere DCE 2001; MP PET 2001; UPSEAT 1999, 2001; AFMC 2002]
4
1 (a) PT  constant (b) PT 1  constant
6. A monoatomic gas (  5 / 3) is suddenly compressed to of its
8 (c) P 1T   constant (d) P1 T   constant
original volume adiabatically, then the pressure of the gas will
change to [CPMT 1976, 83; MP PMT 1994;
13. The amount of work done in an adiabatic expansion from
DPMT 1996; Roorkee 2000; KCET 2000; Pb. PMT 1999, 2001]
temperature T to T1 is [MP PMT 1989]

24 R
(a) (a) R(T  T1 ) (b) (T  T1 )
5  1
(b) 8 (c) RT (d) R(T  T1 )(  1)
40
(c) 14. During the adiabatic expansion of 2 moles of a gas, the internal
3 energy of the gas is found to decrease by 2 joules, the work done
(d) 32 times its initial pressure during the process on the gas will be equal to
7. The pressure and density of a diatomic gas (  7 / 5) change [CPMT 1988]
d' P' 
adiabatically from (P, d) to (P', d'). If  32 , then (a) 1 J
should be[CPMT 1982; EAMCET 2001] (b) 1 J
d P
(a) 1/128 (b) 32 (c) 2 J (d) – 2 J
(c) 128 (d) None of the above 15. The adiabatic elasticity of hydrogen gas (  1.4 ) at NTP is
8 [MP PMT 1990]
8. An ideal gas at 27 o C is compressed adiabatically to of its
27 8
(a) 1  10 N / m
5 2
(b) 1  10 N / m 
5
original volume. If   , then the rise in temperature is[CPMT 1984; CBSE PMT 1999; DPMT 2000;
3 (c) 1.4 N / m 2 (d) 1.4  10 5 N / m 2
BHU 2001; Pb. PET 2001; UPSEAT 2002, 03; KCET 2003;]
16. If  denotes the ratio of two specific heats of a gas, the ratio of
(a) 450 K (b) 375 K slopes of adiabatic and isothermal PV curves at their point of
(c) 225 K (d) 405 K intersection is
9. Two identical samples of a gas are allowed to expand (i) isothermally [NCERT 1990; MH CET 1999; MP PMT 2000]
(ii) adiabatically. Work done is [MNR 1998] (a) 1 /  (b) 
(a) More in the isothermal process (c)  1 (d)   1
(b) More in the adiabatic process
17. Air in a cylinder is suddenly compressed by a piston, which is then
(c) Neither of them maintained at the same position. With the passage of time [NCERT 1971; DPMT 19
(d) Equal in both processes KCET 2000; AIIMS 2000; MH CET 2001]

10. Which is the correct statement [MP PMT 1993] (a) The pressure decreases
(b) The pressure increases
(a) For an isothermal change PV = constant
(c) The pressure remains the same
(b) In an isothermal process the change in internal energy must be
equal to the work done (d) The pressure may increase or decrease depending upon the

nature of the gas
P2  V2 
(c) For an adiabatic change    , where  is the ratio of 18. When a gas expands adiabatically [CPMT 1990]
P1  V1  (a) No energy is required for expansion
specific heats
(b) Energy is required and it comes from the wall of the container
(d) In an adiabatic process work done must be equal to the heat of the gas
entering the system (c) Internal energy of the gas is used in doing work
11. The slopes of isothermal and adiabatic curves are related as
(d) Law of conservation of energy does not hold
[CPMT 1971; BHU 1996; MH CET 1999;
19. One gm mol of a diatomic gas (  1.4 ) is compressed
UPSEAT 2000; RPET 2003]
adiabatically so that its temperature rises from 27 o C to 127 o C .
(a) Isothermal curve slope = adiabatic curve slope The work done will be
(b) Isothermal curve slope =   adiabatic curve slope (a) 2077.5 joules (b) 207.5 joules
(c) Adiabatic curve slope =   isothermal curve slope (c) 207.5 ergs (d) None of the above
664 Thermodynamics
20. Compressed air in the tube of a wheel of a cycle at normal (c) R(T1  T2 ) (d) Zero
temperature suddenly starts coming out from a puncture. The air
inside [NCERT 1970] 29. Helium at 27 o C has a volume of 8 litres. It is suddenly
(a) Starts becoming hotter compressed to a volume of 1 litre. The temperature of the gas will be
(b) Remains at the same temperature [  5 / 3]
(c) Starts becoming cooler [CBSE PMT 1993; MP PMT 1999; Pb. PMT 2002]
(d) May become hotter or cooler depending upon the amount of
water vapour present (a) 108 o C (b) 9327o C
21. The adiabatic Bulk modulus of a perfect gas at pressure is given by[CPMT 1982; MH(c)
CET 2001] o
1200 C (d) 927 o C
(a) P (b) 2P
30. A cycle tyre bursts suddenly. This represents an
(c) P/2 (d) P [SCRA 1994]
22. An adiabatic process occurs at constant (a) Isothermal process (b) Isobaric process
[MNR 1985; AFMC 1996; AIIMS 1999;
UPSEAT 1999, 2000; Pb. PET 2004] (c) Isochoric process (d) Adiabatic process
(a) Temperature 31. One mole of helium is adiabatically expanded from its initial state
(b) Pressure (Pi , Vi , Ti ) to its final state (Pf , Vf , Tf ) . The decrease in the
(c) Heat internal energy associated with this expansion is equal to
(d) Temperature and pressure [SCRA 1994; BHU 2002]
 4 1 (a) CV (Ti  Tf ) (b) CP (Ti  Tf )
23. A polyatomic gas     is compressed to of its volume
 3 8
adiabatically. If its initial pressure is Po , its new pressure will be[MP PET 1994; BHU(c)1995]1 (C  C )(Ti  T ) (d) (CP  CV )(Ti  Tf )
P V f
2
(a) 8 Po (b) 16P0
32. At N.T.P. one mole of diatomic gas is compressed adiabatically to
(c) 6 Po (d) 2 Po
half of its volume   1.41 . The work done on gas will be
 Cp 
24. For adiabatic processes     (a) 1280 J (b) 1610 J
 Cv 
(c) 1815 J (d) 2025 J
[KCET 1999; MP PET 1995; CPMT 2003]
 33. For adiabatic process, wrong statement is [RPMT 1997]
(a) P V = constant (b) T  V = constant
(c) TV  1 =constant (d) TV  = constant (a) dQ  0 (b) dU  dW
25. An ideal gas is expanded adiabatically at an initial temperature of (c) Q = constant (d) Entropy is not constant
300 K so that its volume is doubled. The final temperature of the
34. A diatomic gas initially at 18°C is compressed adiabatically to one-
hydrogen gas is (  1.40) eighth of its original volume. The temperature after compression will
[MP PMT 1995; DPMT 1999] be
(a) 227.36 K (b) 500.30 K [Pb. PET 1995; CBSE PMT 1996; CPMT 1999]
o
(a) 10 C (b) 887 o C
(c) 454.76 K (d)  47 o C
26. A given system undergoes a change in which the work done by the (c) 668 K (d) 144 o C
system equals the decrease in its internal energy. The system must 35. A gas is being compressed adiabatically. The specific heat of the gas
have undergone an during compression is [SCRA 1996]
[Haryana CEE 1996; UPSEAT 2003] (a) Zero (b) Infinite
(c) Finite but non-zero (d) Undefined
(a) Isothermal change (b) Adiabatic change
36. The process in which no heat enters or leaves the system is termed
(c) Isobaric change (d) Isochoric change as [Pb. PET 1996; BHU 1998; BCECE 2003]
27. During the adiabatic expansion of 2 moles of a gas, the internal (a) Isochoric (b) Isobaric
energy was found to have decreased by 100 J. The work done by the (c) Isothermal (d) Adiabatic
gas in this process is [MP PET 1996, 97]
37. Two moles of an ideal monoatomic gas at 27 o C occupies a
(a) Zero (b) –100 J
volume of V. If the gas is expanded adiabatically to the volume 2V ,
(c) 200 J (d) 100 J then the work done by the gas will be
28. In an adiabatic expansion of a gas initial and final temperatures [  5 / 3, R  8.31 J / mol K] [RPET 1999]
are T1 and T2 respectively, then the change in internal energy of
(a) 2767.23 J (b) 2767.23 J
the gas is [MP PET 1997]
(c) 2500 J (d) 2500 J
R R
(a) (T2  T1 ) (b) (T1  T2 )
 1  1
Thermodynamics 665

38. At 27 o C a gas is suddenly compressed such that its pressure (c) (T  4 ) K (d) (T  4 ) K
1 47. A gas is suddenly compressed to 1/4 th of its original volume at
becomes th of original pressure. Temperature of the gas will be
8 normal temperature. The increase in its temperature is (  1.5)
(  5 / 3) [BHU 2000] (a) 273 K (b) 573 K
o (c) 373 K (d) 473 K
(a) 420K (b) 327 C
48. A gas (   1.3) is enclosed in an insulated vessel fitted with
(c) 300 K (d)  142 o C
insulating piston at a pressure of 10 5 N / m 2 . On suddenly
39. U  W  0 is valid for [RPMT 2000]
pressing the piston the volume is reduced to half the initial volume.
(a) Adiabatic process (b) Isothermal process The final pressure of the gas is [RPET 2002]
(c) Isobaric process (d) Isochoric process
(a) 2 0.7  10 5 (b) 2 1.3  10 5
40. An ideal gas at a pressures of 1 atmosphere and temperature of
27 o C is compressed adiabatically until its pressure becomes 8
(c) 2 1.4  10 5 (d) None of these
49. The
times the initial pressure, then the final temperature is ( = 3/2) [EAMCET (Engg.)2000] internal energy of the gas increases In
[MP PMT 1989; RPMT 2001]
(a) 627 o C (b) 527 o C (a) Adiabatic expansion (b) Adiabatic compression
(c) 427 o C (d) 327 o C (c) Isothermal expansion (d) Isothermal compression
50. We consider a thermodynamic system. If U represents the increase
41. Air is filled in a motor tube at 27 o C and at a pressure of 8 in its internal energy and W the work done by the system, which of
atmospheres. The tube suddenly bursts, then temperature of air is the following statements is true
[Given  of air  1.5] [MP PMT 2002] [CBSE PMT 1998]
(a) 27.5 o C (b) 75 o K (a) U  W in an adiabatic process
(b) U  W in an isothermal process
(c) 150 K (d) 150 o C
(c) U  W in an isothermal process
1
42. If   2.5 and volume is equal to times to the initial volume (d) U  W in an adiabatic process
8
51. A gas is suddenly compressed to one fourth of its original volume.
then pressure P' is equal to (Initial pressure = P)
What will be its final pressure, if its initial pressure is P
[RPET 2003]
[Pb. PET 2002]
(a) P'  P (b) P'  2 P
(a) Lesss than P (b) More than P
(c) P'  P  (2)15 / 2 (d) P'  7 P (c) P (d) Either (a) or (c)
43. In an adiabatic process, the state of a gas is changed from 1
P1 , V1 , T1 , to P2 , V2 , T2 . Which of the following relation is 52. A gas for which   1.5 is suddenly compressed to th of the
4
correct [Orissa JEE 2003] initial volume. Then the ratio of the final to the initial pressure is [EAMCET 200
 1  1  1  1 (a) 1 : 16 (b) 1 : 8
(a) T1 V1  T2 V2 (b) P1 V1  P2 V2
(c) 1 : 4 (d) 8 : 1
 
(c) T1 P1  T2 P2 (d) T1 V1   T2 V2 
53. One mole of an ideal gas with   1.4 , is adiabatically compressed
44. During an adiabatic process, the pressure of a gas is found to be so that its temperature rises from 27°C to 35°C. The change in the
proportional to the cube of its absolute temperature. The ratio internal energy of the gas is (R  8.3 J /mol.K) [EAMCET 2001]
C p / Cv for the gas is [AIEEE 2003]
(a) –166 J (b) 166 J
3 4 (c) –168 J (d) 168 J
(a) (b)
2 3
1
54. The volume of a gas is reduced adiabatically to of its volume at
5 4
(c) 2 (d)
3 27°C, if the value of   1.4, then the new temperature will be
45. In adiabatic expansion of a gas
(a) 350  4 0.4 K (b) 300  4 0.4 K
[BCECE 2001; MP PET 2003]
(a) Its pressure increases (c) 150  4 0.4 K (d) None of these
(b) Its temperature falls 55. During an adiabatic expansion of 2 moles of a gas, the change in
(c) Its density increases internal energy was found –50J. The work done during the process
is [Pb. PET 1996]
(d) Its thermal energy increases
46. One mole of an ideal gas at an initial temperature of T K does 6 R (a) Zero (b) 100J
joules of work adiabatically. If the ratio of specific heats of this gas (c) – 50J (d) 50J
at constant pressure and at constant volume is 5/3, the final
temperature of gas will be 56. Adiabatic modulus of elasticity of a gas is 2.1  10 5 N / m 2 . What
[CBSE PMT 2004]  Cp 
will be its isothermal modulus of elasticity   1 . 4 
(a) (T  2.4 ) K (b) (T  2.4 ) K  v
C 
666 Thermodynamics
[UPSEAT 1999] (a) Zero (b) P(V2  V1 )
(a) 1.8  10 N / m
5 2
(b) 1.5  10 N / m 2
5
(c) L  P(V2  V1 ) (d) L
(c) 1.4  10 5 N / m 2 (d) 1.2  10 5 N / m 2
8. A gas expands 0.25m 3 at constant pressure 10 3 N / m 2 , the
P
57. For an adiabatic expansion of a perfect gas, the value of is work done is
P [CPMT 1997; UPSEAT 1999; JIPMER 2001, 02]
equal to [CPMT 1983; MP PMT 1990]
(a) 2.5 ergs (b) 250 J
V V
(a)   (b)  (c) 250 W (d) 250 N
V V
V V 9. Two kg of water is converted into steam by boiling at atmospheric
(c)   (d)   2 pressure. The volume changes from 2  10 3 m 3 to 3.34 m 3 .
V V
The work done by the system is about [
Isobaric and Isochoric Processes (a) – 340 kJ (b) – 170 kJ
(c) 170 kJ (d) 340 kJ
1. A gas expands under constant pressure P from volume V1 to V2 .
The work done by the gas is 10. An ideal gas has volume V0 at 27 o C. It is heated at constant
[CBSE PMT 1990; RPMT 2003] pressure so that its volume becomes 2 V0 . The final temperature is
(a) P(V2  V1 ) (b) P(V1  V2 ) [BCECE 2003]

V1 V2 (a) 54 o C (b) 32.6 o C


(c) P(V1  V2 ) (d) P
V2  V1 (c) 327 C (d) 150 K
2. When heat in given to a gas in an isobaric process, then
[DPMT 2001]
11. If 300 ml of a gas at 27 o C is cooled to 7 o C at constant
pressure, then its final volume will be
(a) The work is done by the gas
[Pb. PET 1999; BHU 2003; CPMT 2004]
(b) Internal energy of the gas increases
(a) 540 ml (b) 350 ml
(c) Both (a) and (b)
(c) 280 ml (d) 135 ml
(d) None from (a) and (b)
12. Which of the following is correct in terms of increasing work done
3. One mole of a perfect gas in a cylinder fitted with a piston has a for the same initial and final state [RPMT 1996]
pressure P, volume V and temperature T. If the temperature is
increased by 1 K keeping pressure constant, the increase in volume (a) Adiabatic < Isothermal < Isobaric
is (b) Isobaric < Adiabatic < Isothermal
2V V (c) Adiabatic < Isobaric < Isothermal
(a) (b)
273 91 (d) None of these
V 13. A sample of gas expands from volume V1 to V2 . The amount of work
(c) (d) V
273 done by the gas is greatest when the expansion is
4. A gas is compressed at a constant pressure of 50 N / m 2 from a [CBSE PMT 1997; AIIMS 1998; JIPMER 2000]
3 3
volume of 10m to a volume of 4m . Energy of 100 J then added (a) Isothermal (b) Isobaric
to the gas by heating. Its internal energy is (c) Adiabatic (d) Equal in all cases
[MNR 1994] 14. Which of the following is a slow process [J & K CET 2000]
(a) Increased by 400 J (b) Increased by 200 J (a) Isothermal (b) Adiabatic
(c) Increased by 100 J (d) Decreased by 200 J (c) Isobaric (d) None of these
5. Work done by air when it expands from 50 litres to 150 litres at a 15. How much work to be done in decreasing the volume of and ideal
constant pressure of 2 atmosphere is
gas by an amount of 2.4  10 4 m 3 at normal temperature and
(a) 2  10 4 joules (b) 2  100 joules constant normal pressure of 1  10 5 N / m 2
(c) 2  10 5  100 joules (d) 2  10 5  100 joules [UPSEAT 1999]
o (a) 28 joule (b) 27 joule
6. Work done by 0.1 mole of a gas at 27 C to double its volume at
constant pressure is (R = 2 cal mol C )
–1 o –1 (c) 25 joule (d) 24 joule
[EAMCET 1994] 16. A Container having 1 mole of a gas at a temperature 27° C has a
movable piston which maintains at constant pressure in container of
(a) 54 cal (b) 600 cal 1 atm. The gas is compressed until temperature becomes 127° C. The
(c) 60 cal (d) 546 cal work done is (C for gas is 7.03 cal/mol K)
p

7. Unit mass of a liquid with volume V1 is completely changed into a (a) 703 J (b) 814 J
gas of volume V2 at a constant external pressure P and (c) 121 J (d) 2035 J
temperature T. If the latent heat of evaporation for the given mass is 17. In a reversible isochoric change [NCERT 1990]
L, then the increase in the internal energy of the system is [Roorkee 1999]
Thermodynamics 667

(a) W  0 (b) Q  0 4. In a cyclic process, the internal energy of the gas

(c) T  0 (d) U  0 (a) Increases (b) Decreases


18. Entropy of a thermodynamic system does not change when this (c) Remains constant (d) Becomes zero
system is used for [AIIMS 1995] 5. Irreversible process is
(a) Conduction of heat from a hot reservoir to a cold reservoir (a) Adiabatic process
(b) Conversion of heat into work isobarically
(b) Joule-Thomson expansion
(c) Conversion of heat into internal energy isochorically
(d) Conversion of work into heat isochorically (c) Ideal isothermal process
19. The work done in which of the following processes is zero (d) None of the above
[UPSEAT 2003] 6. For a reversible process, necessary condition is
(a) Isothermal process (b) Adiabatic process (a) In the whole cycle of the system, the loss of any type of heat
(c) Isochoric process (d) None of these energy should be zero
20. In which thermodynamic process, volume remains same (b) That the process should be too fast
[Orissa PMT 2004] (c) That the process should be slow so that the working substance
should remain in thermal and mechanical equilibrium with the
(a) Isobaric (b) Isothermal surroundings
(c) Adiabatic (d) Isochoric (d) The loss of energy should be zero and it should be quasistatic
21. In an isochoric process if T1  27 o C and T2  127 o C, then 7. In a cyclic process, work done by the system is [BHU 2002]
P1 / P2 will be equal to [RPMT 2003] (a) Zero
(a) 9 / 59 (b) 2 / 3 (b) Equal to heat given to the system
(c) 3 / 4 (d) None of these (c) More than the heat given to system
22. Which is incorrect [DCE 2001] (d) Independent of heat given to the system
8. An ideal gas heat engine operates in a Carnot's cycle between
(a) In an isobaric process, p  0
227 o C and 127 o C . It absorbs 6 × 10 J at high temperature. The
4

(b) In an isochoric process, W  0 amount of heat converted into work is ....


(c) In an isothermal process, T  0 [KCET 2004]
(d) In an isothermal process, Q  0
(a) 4.8  10 4 J (b) 3.5  10 4 J
23. Which relation is correct for isometric process
[RPMT 2001; BCECE 2003] (c) 1.6  10 4 J (d) 1.2  10 4 J

(a) Q  U (b) W  U 9. An ideal heat engine exhausting heat at 77 o C is to have a 30%


efficiency. It must take heat at [BCECE 2004]
(c) Q  W (d) None of these
(a) 127 o C (b) 227 o C
Heat Engine, Refrigerator and (c) 327 o C (d) 673 o C
Second Law of Thermodynamics
10. Efficiency of Carnot engine is 100% if [Pb. PET 2000]
1. A Carnot engine working between 300 K and 600K has work (a) T2  273 K (b) T2  0 K
output of 800 J per cycle. What is amount of heat energy supplied
(c) T1  273 K (d) T1  0 K
to the engine from source per cycle
[DPMT 1999; Pb. PMT 2002, 05; Kerala PMT 2004] 11. A Carnot's engine used first an ideal monoatomic gas then an ideal
(a) 1800 J/cycle (b) 1000 J/cycle diatomic gas. If the source and sink temperature are 411 o C and
(c) 2000 J/cycle (d) 1600 J/cycle 69 o C respectively and the engine extracts 1000 J of heat in each
cycle, then area enclosed by the PV diagram is
2. The coefficient of performance of a Carnot refrigerator working
between 30 o C and 0 o C is [UPSEAT 2002] (a) 100 J (b) 300 J

(a) 10 (b) 1 (c) 500 J (d) 700 J


(c) 9 (d) 0 12. A Carnot engine absorbs an amount Q of heat from a reservoir at
an abosolute temperature T and rejects heat to a sink at a
3. If the door of a refrigerator is kept open, then which of the
following is true [DPMT 2001; BHU 2001;
temperature of T/3. The amount of heat rejected is
JIPMER 2002; AIEEE 2002; CPMT 2003] [UPSEAT 2004]

(a) Room is cooled (a) Q / 4 (b) Q / 3


(b) Room is heated (c) Q / 2 (d) 2Q / 3

(c) Room is either cooled or heated 13. The temperature of sink of Carnot engine is 27 o C . Efficiency of
(d) Room is neither cooled nor heated engine is 25%. Then temperature of source is
668 Thermodynamics
[DCE 2002; CPMT 2002] (c) Reach absolute zero temperature
(a) o
227 C (b) 327 C o (d) Eliminate friction
24. “Heat cannot by itself flow from a body at lower temperature to a
(c) 127 o C (d) 27 o C body at higher temperature” is a statement or consequence of [AIEEE 2003, EAM
14. The temperature of reservoir of Carnot's engine operating with an (a) Second law of thermodynamics
efficiency of 70% is 1000K. The temperature of its sink is (b) Conservation
[DCE 2003] of momentum
(a) 300 K (b) 400 K (c) Conservation of mass
(d) First law of thermodynamics
(c) 500 K (d) 700 K
25. A Carnot engine takes 3  10 6 cal . of heat from a reservoir at
15. In a Carnot engine, when T2  0 o C and T1  200 o C, its
627°C, and gives it to a sink at 27° C. The work done by the engine
efficiency is  1 and when T1  0 o C and T2  200 o C , Its is [AIEEE 2003]
efficiency is  2 , then what is 1 /  2 [DCE 2004] (a) 4.2  10 6 J (b) 8.4  10 6 J
(a) 0.577 (b) 0.733 (c) 16.8  10 J 6
(d) Zero
(c) 0.638 (d) Can not be calculated 26. The first operation involved in a Carnot cycle is
16. The efficiency of Carnot's engine operating between reservoirs, [AFMC 1998]
maintained at temperatures 27 o C and  123 o C, is (a) 2002,
[DPMT Isothermal
03; BVP expansion
2004] (b) Adiabatic expansion
(a) 50% (b) 24% (c) Isothermal compression (d) Adiabatic compression
(c) 0.75% (d) 0.4% 27. For which combination of working temperatures the efficiency of
Carnot’s engine is highest [KCET 2000]
17. A Carnot engine operates between 227 o C and 27 o C. Efficiency (a) 80 K, 60 K (b) 100 K, 80 K
of the engine will be [DCE 1999; BHU 2004] (c) 60 K, 40 K (d) 40 K, 20 K
1 2 28. The efficiency of Carnot engine when source temperature is T and
(a) (b) 1

3 5 sink temperature is T will be


2
[DCE 2000]

3 3 T1  T2 T2  T1
(c) (d) (a) (b)
4 5 T1 T2
18. A measure of the degree of disorder of a system is known as
T1  T2 T1
[Pb. PET 1997; MH CET 1999] (c) (d)
T2 T2
(a) Isobaric (b) Isotropy
(c) Enthalpy (d) Entropy 29. An ideal heat engine working between temperature T and T has an
1 2

19. A carnot engine has the same efficiency between 800 K to 500 K efficiency , the new efficiency if both the source and sink
and x K to 600 K. The value of x is temperature are doubled, will be [DPMT 2000]
[Pb. PMT 1996; CPMT 1996] 
(a) (b) 
(a) 1000 K (b) 960 K 2
(c) 846 K (d) 754 K (c) 2 (d) 3
20. A scientist says that the efficiency of his heat engine which operates 30. An ideal refrigerator has a freezer at a temperature of 13C. The
at source temperature 127°C and sink temperature 27°C is 26%, coefficient of performance of the engine is 5. The temperature of the
then [CBSE PMT 2001] air (to which heat is rejected) will be
(a) It is impossible [BHU 2000; CPMT 2002]
(b) It is possible but less probable (a) 325°C (b) 325K
(c) 39°C (d) 320°C
(c) It is quite probable
31. In a mechanical refrigerator, the low temperature coils are at a
(d) Data are incomplete temperature of – 23°C and the compressed gas in the condenser has
21. A Carnot’s engine is made to work between 200°C and 0°C first and a temperature of 27°C. The theoretical coefficient of performance is
then between 0°C and –200°C. The ratio of efficiencies of the engine [UPSEAT 2001]
in the two cases is [KCET 2002] (a) 5 (b) 8
(c) 6 (d) 6.5
(a) 1.73 : 1 (b) 1 : 1.73
32. An engine is supposed to operate between two reservoirs at
(c) 1 : 1 (d) 1 : 2 temperature 727°C and 227°C. The maximum possible efficiency of
22. Efficiency of a Carnot engine is 50% when temperature of outlet is such an engine is [UPSEAT 2005]
500 K. In order to increase efficiency up to 60% keeping (a) 1/2 (b) 1/4
temperature of intake the same what is temperature of outlet [CBSE PMT 2002] (c) 3/4 (d) 1
(a) 200 K (b) 400 K 33. An ideal gas heat engine operates in Carnot cycle between 227° C
(c) 600 K (d) 800 K and 127°C. It absorbs 6  10 4 cals of heat at higher temperature.
23. Even Carnot engine cannot give 100% efficiency because we cannot[AIEEE 2002] Amount of heat converted to work is
(a) Prevent radiation [CBSE PMT 2005]
(b) Find ideal sources (a) 2.4  10 4 cal (b) 6  10 4 cal
Thermodynamics 669

(c) 1.2  10 4 cal (d) 4.8  10 4 cal L2 L 


2/3

34. Which of the following processes is reversible (c) (d)  2 


[CBSE PMT 2005]
L1  L1 
(a) Transfer of heat by radiation 6. A closed hollow insulated cylinder is filled with gas at 0 o C and
(b) Electrical heating of a nichrome wire also contains an insulated piston of negligible weight and negligible
(c) Transfer of heat by conduction thickness at the middle point. The gas on one side of the piston is
(d) Isothermal compression
heated to 100 o C. If the piston moves 5 cm , the length of the
hollow cylinder is [EAMCET 2001]
(a) 13.65 cm (b) 27.3 cm
(c) 38.6 cm (d) 64.6 cm
7. A mono atomic gas is supplied the heat Q very slowly keeping the
pressure constant. The work done by the gas will be
1. When an ideal diatomic gas is heated at constant pressure, the
fraction of the heat energy supplied which increases the internal 2 3
(a) Q (b) Q
energy of the gas, is 3 5
[IIT 1990; UPSEAT 1998; RPET 2000]
2 1
(c) Q (d) Q
2 3 5 5
(a) (b)
5 5 8. A gas mixture consists of 2 moles of oxygen and 4 moles argon at
3 5 temperature T. Neglecting all vibrational modes, the total internal
(c) (d) energy of the system is
7 7
[IIT 1999; UPSEAT 2003]
2. 1cm 3 of water at its boiling point absorbs 540 calories of heat to
(a) 4 RT (b) 15 RT
become steam with a volume of 1671cm 3 .If the atmospheric
(c) 9 RT (d) 11 RT
pressure = 1.013 x10 5 N / m 2 and the mechanical equivalent of
heat = 4.19 J / calorie, the energy spent in this process in 9. An ideal gas expands isothermally from a volume V1 to V2 and
overcoming intermolecular forces is then compressed to original volume V1 adiabatically. Initial pressure
[MP PET 1999, 2001; Orissa JEE 2002] is P1 and final pressure is P3 . The total work done is W. Then [
(a) 540 cal (b) 40 cal
(a) P3  P1 , W  0 (b) P3  P1 , W  0
(c) 500 cal (d) Zero
3. During the melting of a slab of ice at 273 K at atmospheric pressure (c) P3[IIT 1998]
P1 , W  0 (d) P3  P1 , W  0
(a) Positive work is done by ice-water system on the atmosphere
10. Work done by a system under isothermal change from a volume V1
(b) Positive work is done on the ice-water system by the
atmosphere to V2 for a gas which obeys Vander Waal's equation
(c) The internal energy of the ice-water system increases
 n 2 
(d) The internal energy of the ice-water system decreases (V  n)  P    nRT

4. Two identical containers A and B with frictionless pistons contain  V 
the same ideal gas at the same temperature and the same volume V.
The mass of the gas in A is m A and that in B is m B . The gas in  V  n   V  V2 
(a) nRT loge  2    n2  1
  VV


each cylinder is now allowed to expand isothermally to the same  V1  n   1 2 
final volume 2V. The changes in the pressure in A and B are found
to be P and 1.5 P respectively. Then [IIT 1998]  V     V  V2 
(b) nRT log10  2    n2  1
  VV 

(a) 4 m A  9m B (b) 2m A  3m B  1
V     1 2 
(c) 3m A  2m B (d) 9m A  3m B
 V  n   V  V2 
(c) nRT loge  2    n2  1
  VV 

5. A monoatomic ideal gas, initially at temperature T1 , is enclosed in  1
V  n    1 2 
a cylinder fitted with a frictionless piston. The gas is allowed to
expand adiabatically to a temperature. T 2 by releasing the piston  V  n   VV 
(d) nRT loge  1    n2  1 2
 V V


suddenly. If L1 and L 2 are the lengths of the gas column before  V2  n    1 2 
and after expansion respectively, then T1 / T2 is given by 11. A cylindrical tube of uniform cross-sectional area A is fitted with
[IIT-JEE (Screening) 2000] two air tight frictionless pistons. The pistons are connected to each
2/3 other by a metallic wire. Initially the pressure of the gas is P and
 L1 
0

L1 temperature is T , atmospheric pressure is also P . Now the


(a)   (b) 0 0

 L2  L2 temperature of the gas is increased to 2T , the tension in the wire


0

will be

wire
670 Thermodynamics
(c) 60 % (d) 20 %
19. Which one of the following gases possesses the largest internal
energy [SCRA 1998]

(a) 2 moles of helium occupying 1m 3 at 300 K


(a) 2 P0 A (b) P0 A
(b) 56 kg of nitrogen at 107 Nm 2 and 300 K
P0 A
(c) (d) 4 P0 A (c) 8 grams of oxygen at 8 atm and 300 K
2
12. The molar heat capacity in a process of a diatomic gas if it does a (d) 6  10 26 molecules of argon occupying 40m 3 at 900 K
Q 20. Two samples A and B of a gas initially at the same pressure and
work of when a heat of Q is supplied to it is temperature are compressed from volume V to V/2 (A isothermally
4
and adiabatically). The final pressure of A is
2 5 [MP PET 1996, 99; MP PMT 1997, 99]
(a) R (b) R
5 2 (a) Greater than the final pressure of B
10 6 (b) Equal to the final pressure of B
(c) R (d) R
3 7 (c) Less than the final pressure of B
13. An insulator container contains 4 moles of an ideal diatomic gas at (d) Twice the final pressure of B
temperature T. Heat Q is supplied to this gas, due to which 2 moles 21. Initial pressure and volume of a gas are P and V respectively. First it
of the gas are dissociated into atoms but temperature of the gas is expanded isothermally to volume 4V and then compressed
remains constant. Then adiabatically to volume V. The final pressure of gas will be [CBSE PMT 1999]
(a) Q  2 RT (b) Q  RT (a) 1P (b) 2P
(c) 4P (d) 8P
(c) Q  3 RT (d) Q  4 RT
22. A thermally insulated rigid container contains an ideal gas heated by
14. The volume of air increases by 5% in its adiabatic expansion. The a filament of resistance 100  through a current of 1A for 5 min
percentage decrease in its pressure will be then change in internal energy is
(a) 5% (b) 6% [IIT-JEE (Screening) 2005]
(c) 7% (d) 8% (a) 0 kJ (b) 10 kJ
(c) 20 kJ (d) 30 kJ
15. The temperature of a hypothetical gas increases to 2 times when
compressed adiabatically to half the volume. Its equation can be 23. A reversible engine converts one-sixth of the heat input into work.
written as When the temperature of the sink is reduced by 62°C, the efficiency
of the engine is doubled. The temperatures of the source and sink
(a) PV 3 / 2 = constant (b) PV 5 / 2 = constant are [CBSE PMT 2000]

(c) PV 7 / 3 = constant (d) PV 4 / 3 = constant (a) 80°C, 37°C (b) 95°C, 28°C

16. Two Carnot engines A and B are operated in succession. The first (c) 90°C, 37°C (d) 99°C, 37°C
one, A receives heat from a source at T1  800 K and rejects to 24. An engineer claims to have made an engine delivering 10 kW power
with fuel consumption of 1 g/sec. The calorific value of the fuel is 2
sink at T2 K . The second engine B receives heat rejected by the first kcal/g. Is the claim of the engineer
engine and rejects to another sink at T3  300 K. If the work [J & K CET 2000]
outputs of two engines are equal, then the value of T2 is (a) Valid
(a) 100K (b) 300K (b) Invalid
(c) 550K (d) 700K (c) Depends on engine design
17. When an ideal monoatomic gas is heated at constant pressure, (d) Depends of the load
fraction of heat energy supplied which increases the internal energy
of gas, is [AIIMS 1995]
25. Find the change in the entropy in the following process 100 gm of
ice at 0°C melts when dropped in a bucket of water at 50° C
2 3 (Assume temperature of water does not change) [BHU (Med.) 2000]
(a) (b)
5 5 (a) – 4.5 cal/K (b) + 4.5 cal/K
3 3 (c) +5.4 cal/K (d) – 5.4 cal/K
(c) (d)
7 4 26. An ideal gas expands in such a manner that its pressure and volume
18. When an ideal gas (  5 / 3 ) is heated under constant pressure, can be related by equation PV 2  constant. During this process,
then what percentage of given heat energy will be utilised in doing the gas is [UPSEAT 2002]
external work [RPET 1999] (a) Heated
(a) 40 % (b) 30 % (b) Cooled
Thermodynamics 671
(c) Neither heated nor cooled (d) Relation between U I and U II can not be determined
(d) First heated and then cooled 2. A thermodynamic system is taken through the cycle PQRSP process.
27. A Carnot engine whose low temperature reservoir is at 7°C has an The net work done by the system is
efficiency of 50%. It is desired to increase the efficiency to 70%. By [Orissa JEE 2002]
how many degrees should the temperature of the high temperature (a) 20 J P
reservoir be increased S R
(b) – 20 J 200 Kpa
[UPSEAT 2005]
(c) 400 J
(a) 840 K (b) 280 K 100 Kpa Q
(d) – 374 J P
(c) 560 K (d) 380 K
V
100 cc 300 cc
28. P-V diagram of a diatomic gas is a straight line passing through
origin. The molar heat capacity of the gas in the process will be
(a) 4 R (b) 2.5 R
4R
(c) 3 R (d)
3
29. Following figure shows on adiabatic cylindrical container of volume
V0 divided by an adiabatic smooth piston (area of cross-section =
A) in two equal parts. An ideal gas (CP / CV   ) is at pressure P 1

and temperature T in left part and gas at pressure P and


1 2

temperature T in right part. The piston is slowly displaced and


2

released at a position where it can stay in equilibrium. The final


pressure of the two parts will be (Suppose x = displacement of the
piston)

PT
1 1
PT
2 2

(a) P2 (b) P1
 
V  V 
P1  0  P2  0 
(c)  2  (d)  2 
 
 V0   V0 
  Ax    Ax 
 2   2 
30. Two cylinders A and B fitted with pistons contain equal amounts of
an ideal diatomic gas at 300 K. The piston of A is free to move while
that of B is held fixed. The same amount of heat is given to the gas
in each cylinder. If the rise in temperature of the gas in A is 30 K,
then the rise in temperature of the gas in B is [IIT 1998]
(a) 30 K (b) 18 K
(c) 50 K (d) 42 K

1. A system goes from A to B via two processes I and II as shown in


figure. If U1 and U2 are the changes in internal energies in
the processes I and II respectively, then
P [AIEEE 2005]

(a) UII  UI II

(b) UII  UI A B


I
(c) UI  UII
V
Thermodynemics 673

3. An ideal gas is taken around ABCA as shown in the above P-V 8. P-V plots for two gases during adiabatic process are shown in the
figure. Plots 1 and 2 should correspond respectively to
diagram. The work done during a cycle is [KCET 2001]
P [IIT-JEE (Screening) 2001]
B
(a) 2PV (3P, 3V)
(a) He and O 2 P
(b) PV
(b) O 2 and He
(c) 1/2PV A C 1
(c) He and Ar
(P,V) (P, 3V)
(d) Zero (d) O 2 and N 2 2
V
E D V
4. The P-V diagram shows seven curved paths (connected by vertical 9. Four curves A, B, C and D are drawn in the adjoining figure for a
paths) that can be followed by a gas. Which two of them should be given amount of gas. The curves which represent adiabatic and
parts of a closed cycle if the net work done by the gas is to be at its isothermal changes are
maximum value [CPMT 1986; UPSEAT 1999]
P a [AMU (Engg.) 2000] (a) C and D respectively P
b B C
(a) ac (b) D and C respectively
c
d
(b) cg (c) A and B respectively
e D
f A
(c) af (d) B and A respectively
g
V
(d) cd V 10. In pressure-volume diagram given below, the isochoric, isothermal,
5. An ideal gas of mass m in a state A goes to another state B via three and isobaric parts respectively, are
different processes as shown in figure. If Q1 , Q2 and Q 3 denote [Manipal MEE 1995]
P A B
the heat absorbed by the gas along the three paths, then (a) BA, AD[MP PET 1992]
, DC
P A
(a) Q1  Q2  Q3 (b) DC, CB, BA C
(c) AB, BC, CD
(b) Q1  Q2  Q3 3
2 (d) CD, DA, AB D
V
(c) Q1  Q2  Q3 1
11. The P-V diagram of a system undergoing thermodynamic
B transformation is shown in figure. The work done on the system in
(d) Q1  Q2  Q3
V going from A  B  C is 50 J and 20 cal heat is given to the
6. Which of the following graphs correctly represents the variation of P between A and CC is
system. The change in internal energy
  (dV / dP ) / V with P for an ideal gas at constant (a) 34 J
temperature [IIT-JEE (Screening) 2002] (b) 70 J
(a) (b) (c) 84 J
 
A B
(d) 134 J
V
12. An ideal gas is taken through the cycle A  B  C  A, as shown
in the figure. If the net heat supplied to the gas in the cycle is 5 J,
P P the work done by the gas in the process
3
C  A is
V (m )
(c)  (d) (a) – 5 J C
2 B
(b) – 10 J
(c) – 15 J 1
A
P P
7. A thermodynamic process is shown in the figure. The pressures and (d) – 20 J
10 P(N/m2)
volumes corresponding to some points in the figure are :
13. In the following indicator diagram, the net amount of work done
PA  3  10 4 Pa, PB  8  10 4 Pa and
will be
VA  2  10 3 m 3 , VD  5  10 3 m 3 P
(a) Positive
In process AB, 600 J of heat is added to the system and in process
BC, 200 J of heat is added to the system. The change in internal (b) Negative 1 2
energy of the system in process AC would be [CBSE PMT 1992]
B (c) Zero
(a) 560 J P C
(b) 800 J (d) Infinity V

(c) 600 J 14. A cyclic process for 1 mole of an ideal gas is shown in figure in the
V-T, diagram. The work done in AB, BC and CA respectively
(d) 640 J
A D
O V
674 Thermodynamics
[UPSEAT 1998; RPET 2000; Kerala PMT 2002]
V 
(a) 0, RT2 ln 1  , R (T1  T2 )
 (a) PV (2P, V) (2P, 2V)
 V2  P
V (b) 2PV
C
V1 V2
(b) R(T1  T2 ), 0, RT1 ln (c) PV/2
V2
(P, V) (P, 2V)
(d) 3PV
V 
(c) 0, RT2 ln 2  , R (T1  T2 )
 V1 A
B 19. V
A cyclic process ABCA is shown in the V-T diagram. Process on the
 V1  P-V diagram is
O T
T1 T2
V  V
(d) 0, RT2 ln 2  , R (T2  T1 )

 V1 
C B
15. A cyclic process ABCD is shown in the figure P-V diagram. Which of
the following curves represent the same process
A T
P P C
A B (a) (b) P
B

C A
B A C
V V
D P
V (c) (d) P
A A
B
(a) P A B (b) P A B
C B
C
C C V V
20. In the figure given two processes A and B are shown by which a
D D
thermo-dynamical system goes from initial to final state F. If Q A
T T
(c) P B
(d) P and QB are respectively the heats supplied to the systems then

A A B (a) Q A  QB P
A
C (b) Q A  QB
D D C
i f
16. T of a gas represented by a Pressure-Volume
Carnot cycle (reversible) T (c) Q A  QB
curve is shown in the diagram B
(d) Q A  QB
Consider the following statements
21. In the cyclic process shown in the figure, the work done by the Vgas
I. Area ABCD = Work done on the gas in one cycle is P [MP PMT 1999]
II. Area ABCD = Net heat absorbed 7P 1
(a) 28 P1V1
III. Change in the internal energy in cycle = 0
(b) 14 P1V1
Which of these are correct [AMU (Med.) 2001]
(c) 18 P1V1
(a) I only P
A P1
(b) II only B
(d) 9 P1V1 V
V1 4V1
22. An ideal gas is taken around the cycle ABCA as shown in the P-V
(c) II and III
D diagram. The net work done by the gas during the cycle is equal to
(d) I, II and III C
V (a) 12 P1V1 P
17. The temperature-entropy diagram of a reversible engine cycle is B
(b) 6 P1V1 3P 1
given in the figure. Its efficiency is

T [AIEEE 2005] (c) 3 P1V1


(a) 1 / 3 (d) 2 P1V1 P1 A C
2T0 23. Heat energy absorbed by a system in going through
(b) 2/3 V1 3V1a cyclic process
shown in figure is [AIIMS 1995; BHU 2002]
(c) 1 / 2
T0 (a) 10  J 7 V (litre)
(d) 1 / 4 S 30
S0 2 S0 (b) 10  J 4

18. Work done in the given P-V diagram in the cyclic process is
10

10 30 P (kPa)
Thermodynemics 675

(c) 10  J 2 (a) AB and BC


(b) AB and CD
(d) 10 3 J
(c) BC and DA
24. A thermodynamic system is taken from state A to B along ACB and
(d) BC and CD
is brought back to A along BDA as shown in the PV diagram. The
net work done during the complete cycle is given by the area 29. An ideal monoatomic gas is taken round the cycle ABCDA as shown
[CBSE PMT 1992]
P in following P-V diagram. The work done during the cycle is [IIT 1983; CPMT 199
(a) P ACBP P B P
1 2 1

P2 (3P, V) (3P, 3V)


(b) ACBB'A'A C
(a) PV
(c) ACBDA D
(b) 2 PV
P1 A
(d) ADBB'A'A (c) 4 PV
25. A  volume with
In the diagrams (i) to (iv) of variation of B  changing
V (P, V) (P, 3V)
(d) Zero O V
pressure is shown. A gas is taken along the path ABCD. The change
in internal energy of the gas will be 30. A system changes from the state (P1 , V1 ) to (P2 V2 ) as shown in
[CPMT 1986, 88] the figure. What is the work done by the system
V D V
C D C P(N/m2) [CPMT 1981]

(a) 7.5  10 joule


5
5  105 (P2, V2)

A A B (b) 7.5  10 5 erg


B 1  105
(P1, V1)
(i)
(c) 12  10 5 joule
P (ii) P
D C D
V V (d) 6  10 5 joule
C 1 2 3 4 5 V ( m3 )
31. Carbon monoxide is carried around a closed cycle abc in which bc is
an isothermal process as shown in the figure. The gas absorbs 7000
A J of heat as its temperature increases from 300 K to 1000 K in going
A B B from a to b. The quantity of heat rejected by the gas during the
process ca is
P (i) to (iv) (iv) P P
(a) Positive
(iii) in all cases
b [SCRA 1994]
P2
(b) Positive in cases (i), (ii) and (iii) but zero in (iv) case (a) 4200 J
(c) Negative in cases (i), (ii) and (iii) but zero in (iv) case (b) 5000 J
(d) Zero in all four cases a c
(c) 9000 J P1
26. A system is taken through a cyclic process represented by a circle as
shown. The heat absorbed by the system is (d) 9800 J V
V1 V2
32. A sample of ideal monoatomic gas is taken round the cycle ABCA as
(a)   10 3 J V(in cc)
shown in the figure. The work done during the cycle is [
40
 P B
(b) J 30
2 (a) Zero (4P, 3V)
20 (b) 3 PV
(c) 4  10 2 J
0 (c) 6 PV
(d)  J 0 50 100 150 200 P(in kPa) A C
(d) 9PV (P , V ) ( P , 3V )
27. A thermodynamic system undergoes cyclic process ABCDA as shown V
in figure. The work done byP the system is 33. When a system is taken from state i to a state f along path iaf,
C B
3P 0 Q  50 J and W  20J . Along path ibf, Q  35 J . If
(a) P0 V0
W  13 J for the curved return path f i, Q for this path is
(b) 2 P0 V0 2P0 O [AMU (Med.) 2000]
P
P0 V0 a
(c) P0 (a) 33 J f
2 A D
V (b) 23 J
(d) Zero V0 2V0
(c) – 7 J
28. The P-V graph of an ideal gas cycle is shown here as below. The i b
(d) – 43 J
adiabatic process is described by V
34. For one complete cycle of a thermodynamic process on a gas as
[CPMT 1985; UPSEAT 2003]
shown in the P-V diagram, Which of following is correct
P A
P
B

D
C
V
676 Thermodynamics

(a) Eint  0, Q  O (a) (b)


P
(b) Eint  0, Q  0

(c)  Eint  0, Q  0
(d)  Eint  0, Q  0 V
(c) P (d) P
35. An ideal gas is taken around ABCA as shown in the above P-V
diagram. The work done during a cycle is
[UPSEAT 2001]
(a) Zero P C
(2P, 3V) V V
1 41. In the following figure, four curves A, B, C and D are shown. The
(b) PV
2 curves are [DCE 2003]

(c) 2 PV A B P P
(P , V ) ( P , 3V )
(d) PV O V A
36. An ideal gas is taken from point A to the point B, as shown in the B
P-V diagram, keeping the temperature constant. The work done in C
the process is [UPSEAT 2005] D
(a) (PA  PB )(VB  VA ) P V V
(a) Isothermal for A and D while adiabatic for B and C
1 A
(b) (PB  PA )(VB  VA ) PA (b) Adiabatic for A and C while isothermal for B and D
2
(c) Isothermal for A and B while adiabatic for C and D
1
(c) (PB  PA )(VB  VA )
2 PB B (d) Isothermal for A and C while adiabatic for B and D

1 O V 42. P-V diagram of a cyclic process ABCA is as shown in figure. Choose


(d) (PB  PA )(VB  VA ) VA VB
the correct statement
2
37. The P-V diagram of a system undergoing thermodynamic (a) Q A  B = negative P A
transformation is shown in figure. The work done by the system in
going from A  B  C is 30J and 40J heat is given to the system. (b) UB C = positive
The change in internal energy between A and C is [BCECE 2005]
P (c) WCAB = negative B
(a) 10 J C
C
(d) All of these V
(b) 70 J
43. A sample of an ideal gas is taken through a cycle a shown in figure.
(c) 84 J It absorbs 50J of energy during the process AB, no heat during BC,
A B
(d) 134 J rejects 70J during CA. 40J of work is done on the gas during BC.
V
Internal energy of gas at A is 1500J, the internal energy at C would
38. Consider a process shown in the figure. During this process the
be
work done by the system
P
(a) Continuously increases (a) 1590 J P B
B
(b) Continuously decreases A (b) 1620 J
(c) First increases, then decreases (c) 1540 J
(d) First decreases, then increases V C A
(d) 1570 J
39. Six moles of an ideal gas perfomrs a cycle shown in figure. If the V
temperature are T = 600 K, T = 800 K, T = 2200 K and T = 1200 K,
A B C D
44. In the following P-V diagram two adiabatics cut two isothermals at
the work done per cycle is V
temperatures T and T (fig.). The value of a will be
Vd
1 2

[BCECE 2005]
P B
(a) 20 kJ C
(b) 30 kJ A P
D
(c) 40 kJ
a b T1
(d) 60 kJ
T d c
40. Which of the accompanying PV, diagrams best represents an
T2
isothermal process [MP PET 2005]

P
Va Vd Vb Vc V

V
Thermodynemics 677

Vb Vc Reason : When a system changes from one thermal


(a) (b) equilibrium to another, some heat is absorbed by it.
Vc Vb
12. Assertion : A room can be cooled by opening the door of a
Vd refrigerator in a closed room.
(c) (d) Vb Vc
Va Reason : Heat flows from lower temperature (refrigerator) to
higher temperature (room).
13. Assertion : It is not possible for a system, unaided by an
external agency to transfer heat from a body at
lower temperature to another body at higher
temperature.
Reason : According to Clausius statement, “ No process is
Read the assertion and reason carefully to mark the correct option out of
possible whose sole result is the transfer of heat
the options given below:
from a cooled object to a hotter object.
(a) If both assertion and reason are true and the reason is the correct
explanation of the assertion. 14. Assertion : If an electric fan be switched on in a closed room,
(b) If both assertion and reason are true but reason is not the correct the air of the room will be cooled.
explanation of the assertion. Reason : Fan air decrease the temperature of the room.
(c) If assertion is true but reason is false.
(d) If the assertion and reason both are false. 15. Assertion : The internal energy of an isothermal process does
(e) If assertion is false but reason is true. not change.
1. Assertion : Reversible systems are difficult to find in real world. Reason : The internal energy of a system depends only on
Reason : Most processes are dissipative in nature pressure of the system.
[AIIMS 2005]
16. Assertion : In an adiabatic process, change in internal energy of
2. Assertion : Air quickly leaking out of a balloon becomes coolers a gas is equal to work done on or by the gas in the
Reason : The leaking air undergoes adiabatic expansion.[AIIMS 2005] process.
3. Assertion : Thermodynamic process in nature are irreversible.
Reason : Dissipative effects can not be eliminated. Reason : Temperature of gas remains constant in a adiabatic
[AIIMS 2004] process.
4. Assertion : When a bottle of cold carbonated drink is opened, a 17. Assertion : An adiabatic process is an isoentropic process.
slight fog forms around the opening.
Reason : Change in entropy is zero in case of adiabatic
Reason : Adiabatic expansion of the gas causes lowering of
temperature and condensation of water vapours.[AIIMS 2003] process.
5. Assertion : The isothermal curves intersect each other at a 18. Assertion : Work done by a gas in isothermal expansion is
certain point. more than the work done by the gas in the same
Reason : The isothermal change takes place slowly, so the expansion, adiabatically.
isothermal curves have very little slope. Reason : Temperature remains constant in isothermal
[AIIMS 2001] expansion and not in adiabatic expansion.
6. Assertion : In adiabatic compression, the internal energy and
temperature of the system get decreased. 19. Assertion : First law of thermodynamics is a restatement of the
Reason : The adiabatic compression is a slow process principle of conservation
[AIIMS 2001] Reason : Energy is fundamental quantity.
7. Assertion : In isothermal process whole of the heat energy 20. Assertion : Zeroth law of thermodynamic explain the concept
supplied to the body is converted into internal of energy.
energy.
Reason : According to the first law of thermodynamics Reason : Energy is dependent on temperature.
Q  U  pV . [AIIMS 1997] 21. Assertion : Efficiency of a Carnot engine increase on reducing
8. Assertion : We can not change the temperature of a body the temperature of sink.
without giving (or taking) heat to (or from) it. Reason : The efficiency of a Carnot engine is defined as ratio
Reason : According to principle of conservation of energy, of net mechanical work done per cycle by the gas to
total energy of a system should remains conserved. the amount of heat energy absorbed per cycle from
9. Assertion : The specific heat of a gas is an adiabatic process is the source.
zero and in an isothermal process is infinite. 22. Assertion : The entropy of the solids is the highest
Reason : Specific heat of a gas in directly proportional to Reason : Atoms of the solids are arranged in orderly manner.
change of heat in system and inversely proportional
to change in temperature.
10. Assertion : Work and heat are two equivalent form of energy.
Reason : Work is the transfer of mechanical energy
irrespective of temperature difference, whereas heat
is the transfer of thermal energy because of
temperature difference only.
First Law of Thermodynamics (Q = U + W)
11. Assertion : The heat supplied to a system is always equal to the
increase in its internal energy. 1 a 2 c 3 b 4 b 5 c
678 Thermodynamics

6 b 7 b 8 d 9 a 10 d 21 b 22 b 23 c 24 a 25 b
11 c 12 a 13 d 14 a 15 b 26 a 27 d 28 a 29 b 30 c
16 b 17 c 18 d 19 d 20 b 31 a 32 a 33 c 34 d
21 a 22 d 23 b 24 a 25 d
26 d 27 a 28 b 29 d 30 a Critical Thinking Questions
31 b 32 c 33 c 34 a 35 a
36 b 37 c 38 c 39 b 40 a 1 d 2 c 3 bc 4 c 5 d
41 c 42 a 43 a 44 c 45 c 6 d 7 c 8 d 9 c 10 a
46 a 47 c 48 b
11 b 12 c 13 b 14 c 15 a

Isothermal Process 16 c 17 b 18 a 19 b 20 c
21 b 22 d 23 d 24 b 25 b
1 c 2 a 3 c 4 d 5 b
26 b 27 d 28 c 29 c 30 d
6 b 7 c 8 d 9 a 10 c
11 a 12 b 13 a 14 a 15 c
16 c 17 a 18 c 19 a 20 c
Graphical Questions
21 b 22 b 23 a 24 a 25 a
1 c 2 b 3 a 4 c 5 a
26 c 27 b 28 b 29 b 30 a
6 a 7 a 8 b 9 c 10 d
31 d
11 d 12 a 13 b 14 c 15 a
Adiabatic Process 16 c 17 a 18 a 19 c 20 d
21 d 22 d 23 c 24 c 25 d
1 c 2 c 3 b 4 d 5 c
26 b 27 d 28 c 29 c 30 c
6 d 7 c 8 b 9 a 10 a
31 d 32 b 33 d 34 a 35 d
11 c 12 d 13 b 14 d 15 d
36 d 37 a 38 a 39 c 40 b
16 b 17 a 18 c 19 a 20 c
41 d 42 d 43 a 44 a
21 d 22 c 23 b 24 c 25 a
26 b 27 d 28 a 29 d 30 d Assertion & Reason
31 a 32 c 33 d 34 c 35 a
36 d 37 b 38 d 39 a 40 d 1 a 2 a 3 a 4 a 5 e
6 d 7 e 8 d 9 a 10 a
41 c 42 c 43 a 44 a 45 b
11 d 12 d 13 a 14 d 15 c
46 d 47 a 48 b 49 b 50 a
16 c 17 a 18 b 19 c 20 e
51 b 52 d 53 b 54 b 55 d
21 b 22 a
56 b 57 c

Isobaric and Isochoric Processes

1 a 2 c 3 c 4 a 5 a
6 c 7 c 8 b 9 d 10 c
11 c 12 a 13 b 14 a 15 d
First Law of Thermodynamics (Q = U + W)
16 b 17 a 18 d 19 c 20 d
21 d 22 d 23 a 1. (a) Q  U  W and W  PV
2. (c)
Heat Engine, Refrigerator and 3. (b) Q  U  W
Second Law of Thermodynamics
 U  Q  W  Q  W (using proper sign)

1 d 2 c 3 b 4 c 5 b 4. (b) U  Q  W  35  15  20 J
6 d 7 b 8 d 9 b 10 b 5. (c) Internal energy depends only on the temperature of the gas.
11 c 12 b 13 c 14 a 15 a 6. (b)
16 a 17 b 18 d 19 b 20 a
Thermodynemics 679

7. (b) (i) Case  Volume = constant 


 PdV  0
2 V1 2 V1
(ii) Case  P = constant   V1
PdV  P  V1
dV  PV1

8. (d) Q  W  U  35  15  U  U  50 J
9. (a) JQ  U  W, U  JQ  W

U  4.18  300  600  654 Joule


2
10. (d) Work done   1
PdV , which is state dependent as well as
path dependent.
f
11. (c) Q  U  W  W  0  Q  U  R T
2
3
  2 R(373  273) = 300R.
2
12. (a) Q  2k cal  2  10 3  4.2 J  8400 J and W  500J.
Hence from Q  U  W, W  Q  U = 8400 –
500 = 7900 J
13. (d) Change in internal energy (U) depends upon initial an find
state of the function while Q and W are path dependent
also.
14. (a) This is the case of free expansion and in this case W  0 ,
U  0 so temperature remains same i.e. 300 K.
15. (b) Q  U  W  W  Q  U  100  40  70 J
16. (b) Work done is not a thermodynamical function.
17. (c) Q  U  W  167  333  500 cal
18. (d) Heat always refers to energy in transit from one body to
another because of temperature difference.
19. (d) Change in internal energy does not depend upon path so
U  Q  W remain constant.
680 Thermodynamics

20. (b) Q  U  W ; Q  200 J and W  100 J  12.48  10 2 J


 U  Q  W  200  (100)  300 J  R 
38. (c) U  CV T  n  T
21. (a) During free expansion of a perfect gas no, work is done and   1 
also no heat is supplied from outside. Therefore, no change in
internal energy. Hence, temperature remain constant. PV P(2 V  V ) PV
 U   
22. (d) Q  U  W  U  Q  W  150  110  40 J (  1)  1 (  1)

23. (b) From FLOT Q  U  W 39. (b) U  CV T  2  4.96  (342  340)  19.84 cal
40. (a)
 Heat supplied to the system so Q  Positive
41. (c) According to FLOT
and work is done on the system so W  Negative
Q  U  P(V )  U  Q  P(V )
Hence +Q = U – W
24. (a)  1500  (2.1  10 5 )(2.5  10 3 ) =975 Joule
25. (d) State of a thermodynamic state cannot determine by a single 42. (a) Q  U  W  U  Q  W
variable (P or V or T)
26. (d) R is the universal gas constant.  6  4.18  6  19.08kJ  19.1 kJ

27. (a) From FLOT 43. (a) Given Q  20J , W  8 J and U i  30 J


 dU  dQ  dW  dU  dQ( 0) ( dW  0) Q  U  W  U  (Q  W )
 dU  0 So temperature will decrease.  (U f  Ui ) = (U f  30)  20  (8)  U f  18 J
28. (b) From FLOT Q = U + W 44. (c) Change in internal energy U   CV T
Work done at constant pressure (W )P  (Q)P  U it doesn’t depend upon type of process. Actually it is a state
function
(Q)P  (Q)V (As we know (Q)V  U )
45. (c)
Also (Q)P  mc P T and (Q)V  mc V T 46. (a) In first process using Q  U  W
 (W )P  m(c P  c V )T  8  10 5  U  6.5  10 5  U  1.5  10 J
Since final and initial states are same in both process
 (W )P  1  (3.4  10 3  2.4  10 3 )  10  10 4 cal
So U will be same in both process
29. (d)
For second process using Q  U  W
30. (a) Ideal gas possess only kinetic energy.
31. (b) The internal energy and entropy depend only on the initial and  10 5  1.5  10 5  W  W  0.5  10 5 J
final states of the system and not on the path followed to
attain that state. 47. (c) W  PV ; here V is negative so W will be negative
48. (b) Entropy is related to second law of thermodynamics.
32. (c) Q  U  W
 Q  200cal  200  4.2  840 J and W  40 J Isothermal Process
 U  Q  W  840  40  800 J
1. (c) In isothermal process temperature remains constant.
33. (c) Q  U  W  (U f  Ui )  W
2. (a) If isothermal curves cut each other then at equilibrium two
temperature will be there which is impossible.
 30  (U f  40)  10  U f  60 J
3. (c) In isothermal expansion temperature remains constant, hence
34. (a) With rise in temperature, internal energy also increases. no change in internal energy.
35. (a) V2
36. (b) Heat supplied to a gas raise its internal energy and does some 4. (d) W  RT loge
V1
work against expansion, so it is a special case of law of
conservation of energy. m V m V
   RT loge 2  2.3  RT log10 2
37. (c) Change in internal energy is always equal to the heat supplied M V1 M V1
at constant volume.
96 140
i.e. U  (Q)V  CV T .  2.3  R (273  27) log10  2.3  900 R log10 2
32 70
3
For monoatomic gas CV  R 5. (b) 0.8  5  P  (3  5)  P  0.5 m
2
6. (b) Differentiate PV  constant w.r.t V
3  3
 U    R T  1   8 .31  (100  0) P V
2  2  PV  VP  0  
P V
Thermodynamics 681
7. (c) Atmospheric pressure P  1.01  10 6 dyne / cm 2
V2  22.4 
8. (d) W  RT loge  1  8.31  (273  0) loge   Work done against atmospheric pressure
V1  11.2 
1 .01  10 6  1670
  8.31  273  loge 2  1572.5 J [  loge 2  0.693 ] W  PV   40 cal
4 .2  10 7
9. (a) E  P , if P  constant, E  constant V 20
28. (b) Wiso  RT loge 2  1  8 .31  300 loge  1728 J
RT V1 10
10. (c) For isothermal process PV  RT  P 
V V 
V2 RT V 29. (b) W  RT loge  2   0.2  8.3  loge 2  (27  273)
 W  PdV   V1 V
dV  RT loge 2
V1
 V1 
 0.2  8.3  300  0.693  345 J
11. (a) E  P 30. (a) For isothermal process P1 V1  P2 V2
1 P1 V1 72  1000
12. (b) For such a case, pressure   P2   =80 cm
Compressib ility V2 900
13. (a) E  P  1.013  10 5 N / m 2 Stress P  P2  P1  80  72  8 cm
14. (a) In isothermal process, compressibility E = . 31. (d) During isothermal change T = constant  U = 0
15. (c) In isothermal process, exchange of energy takes place between also from FLOT, Q = W.
system and surrounding to maintain the system temperature
constant. Adiabatic Process
16. (c) No change in the internal energy of ideal gas but for real gas
internal energy increases because work is done against 1. (c) Gas cylinder suddenly explodes is an irreversible adiabatic
intermolecular forces. change and work done against expansion reduces the
temperature.
17. (a) In isothermal process temperature remains constant. i.e.,
R(T1  T2 )
Q 2. (c) Work done in adiabatic change 
T  0 . Hence according to C   Ciso    1
m T
18. (c) This is the case of free expansion of gas. In free expansion 3. (b) In case of adiabatic expansion W = positive and Q  0
U  0  Temp. remains same. from FLOT Q  U  W  U  W i.e.,
19. (a) An isothermal process takes place at constant temperature, U will be negative.
must be carried out in a vessel with conducting wall so that T
heat generated should go out at once. 4. (d) For adiabatic process  constant
P  1
20. (c) For isothermal process
1  (1  1 .4 )
P 
0.4
dU  0 and work done  dW  P(V2  V1 ) T  T2 4 1.4 
 2   1      T2  300(4 ) 1 .4
T1  P2  300  1 
V1 V PV
 V2    dW    3/2
2 2 2 P2  V1  P  V 
5. (c) PV  constant      2   1  8
21. (b) In isothermal process, temperature remains constant. P1  V2  1  V1 / 4 
22. (b) In isothermal process, heat is released by the gas to maintain  P2  8 atm .
the constant temperature.

23. (a) In isothermal compression, there is always an increase of heat. P2  V1 
6. (d) PV  =constant      P2  (8 )5 / 3 P1  32 P1
P1  V2 
which must flow out the gas.
Q  U  W  Q  W ( U  0)
m
7. (c) Volume of the gas V  and using PV  = constant
1.5  104
d
 Q  1.5  10 4 J  cal  3 .6  10 3 cal
4.18  
P'  V   d ' 
We get        (32)7 / 5  128
24. (a) In isothermal change, temperature remains constant, Hence U P  V'   d 
= 0.
 1 5 2
1
Also from Q  U  W  Q  W T2  V1   27  3  27  3
8. (b)    T2  300    300  
T1  V2   8   8 
25. (a) It is an isothermal process. Hence work done  P(V2  V1 )
2

 27  
1/3
 3
2
 1  10 5  (1.091  1)  10 6  0.0091 J  300     800    675 K

 8   2
26. (c) Q  U  W  U  Q  W  2240  168  2072 J . 
 T  675  300  375 K
27. (b) Amount of heat given  540 calories
9. (a) In thermodynamic processes.
Change in volume V  1670 c.c
Work done = Area covered by PV diagram with V-axis
682 Thermodynamics
From graph it is clear that (Area)iso  (Area)adi T2  V1 

V 

25. (a) TV  1  constant     T2  T1  1 
 Wiso  Wadi P T1  V2 

V
 2


0.4
Isothermal 1
 T2  300   227.36 K
Adiabatic 2
V 26. (b) In adiabatic change Q = constant  Q = 0
10. (a) Since PV  RT and T  constant;  PV  constant. So W = – U ( Q = U + W)
11. (c) For Isothermal process PV  constant 27. (d) For adiabatic process from FLOT
 dP   P
   Slope of Isothermal curve W  U ( Q  0)
 dV  V
 W  (100)  100 J
For adiabatic PV   constant
dP P R(T1  T2 ) R(T2  T1 )
   Slop of adiabatic curve slope 28. (a) U  W   
dV V (  1)  1
 dP   dP   1
Clearly,     V 
 dV  adiabatic  dV  Isothermal 29. (d) TV  1  constant  T2  T1  1 
  927 o C
  V2 
 RT 
12. (d) PV  constant  P    constant 30. (d) The process is very fast, so the gas fails to gain or lose heat.
 P 
Hence this process in adiabatic
 P1 T   constant.
31. (a) U  CV T  1  CV (Tf  Ti )   CV (Ti  Tf )
R R
13. (b) Wadi  (Ti  Tf )  (T  T1 )
 1  1  |U| = C (T – T )
V i f

14. (d) dQ  0  2  dW  dW  2 J  1
V 
 Work done by the gas  2 J
32. (c) T2  T1  1 
  273(2)0.41  273  1.328  363 K
 V2 
 Work done on the gas  2 J
R(T1  T2 ) 8 .31(273  363)
W    1824
15. (d) E  P  1.4  (1  10 5 )  1.4  10 5 N / m 2  1 1 .41  1
16. (b) Slope of adiabatic curve =   (Slope of isothermal curve)  |W|  1815 J
17. (a) Due to compression the temperature of the system increases to
a very high value. This causes the flow of heat from system to 33. (d)
the surroundings, thus decreasing the temperature. This
decrease in temperature results in decrease in pressure. 34. (c) TV  1  constant
 1
18. (c) Q  U  W  0  W  U V   V 
0 .4
 T2  T1  1 
  (273  18)    668 K
if W is positive i.e., gas does work then U should be  V2  V /8 
negative meaning internal energy is used in doing work.
35. (a) Q  mc . Here Q  0 , hence c  0
R
19. (a) W (T1  T2 ) 36. (d) In adiabatic process, no transfer of heat takes place between
 1
system and surrounding.
8.31  {( 273  27)  (273  127)}
  2077.5 joules R(T1  T2 ) RT1  T2 
1 .4  1 37. (b) W   1  
(  1) (  1)  T1 
20. (c) Pressure is reduced, so the temperature falls.
 1 
21. (d) Adiabatic Bulk modulus E  P RT1   V1  
 1 
(  1)   V2  
22. (c) In adiabatic process, no heat transfers between system and  
surrounding.  1 
5
2  8 .31  300 1   1  3   2767.23 J
  
P2  V1  V  5   2 
23. (b)    P2  P1  1   P0 (8 )4 / 3  16 P0 .   1  
P1  V2 

V
 2

 3 
 1
24. (c) In adiabatic process PV  constant 38. (d) T P  1 
 constant  T  P 

 1
 RT   5 / 3 1
  .V  constant  TV  1 = constant T P   1 5/3
 V   2   2   
T1  P1  8
Thermodynamics 683

1
0.4
54. (b) For adiabatic change TV  1 = constant
T2  300     131K  142C  1  1
8 T2  V1  V 
    T2   1   T1
39. (a) In adiabatic process Q = 0  U  W  0 T1  V2 
  V2


(Q  U  W ) 1 .4 1
 V 
 1  T2     300  300  (4 )0.4 K
3 / 2 1 V /4 
T2  P2  
40. (d) Using relation    (8 ) 3/2
2. (d) For adiabatic forces W  U ( Q  0)
T1  P1 
55.
 W  (50)  50 J
 T2  2T1  T2  2 (273  27)  600 K  327C
Adiabatic elasticicity (E ) E
 1 1 .5 1 56. (b)    E 

1
T2  P2   T2  1 Isothermal elasticicity (E )
 1 .5 1 1 3
41. (c)        
T1  P1  T1  8  8 2 2 .1  10 5
 E   1.5  10 5 N / m 2
T1 300 1.4
 T2    150 K . 57. (c) PV  constant : Differentiating both sides
2 2
 dP dV
P2  V1  P' PV  1 dV  V  dP  0   
42. (c)     (8 )5 / 2  P '  P  (2)15 / 2 P V
P1  V2 
 P
43. (a) Isobaric and Isochoric Processes
44. (a) Given P  T 3 , but we know for an adiabatic process, the
pressure P  T  /  1 1. (a) Work done  PV  P(V2  V1 )
 3 C 3 2. (c) When heat is supplied at constant pressure, a part of it goes in
So  3    P 
 1 2 CV 2 the expansion of gas and remaining part is used to increase the
temperature of the gas which in turn increases the internal
45. (b) energy.
R(Ti  Tf ) R(T  Tf )
46. (d) W   6R   Tf  (T  4 )K. 3.
V T
(c) For isobaric process 2  2  V2  V 
274
 1 5 
  1 V1 T1 273
3  274 V V
Increase  V 
47. (a)  TV  1  constant T1V1 1  T2 V2 1 273 273
 1 4. (a) From FLOT Q  U  W  U  PV
V 
 T2  T1  1   T1 (4 )1.5 1  2T1  100  U  50  (4  10)  U  400 J
 V2 
5. (a) W  P  V  2  10 5 (150  50)  10 3  2  10 4 J
 change in temperature
6. (c) W  PV  nRT  0.1  2  300  60 cal
 T2  T1  2T1  T1  T1  273 K
7. (c) Q  V  PV  mL = U + P(V – V )
48. (b)  PV  k (constant)  P1V1  P2 V2 2 1

 U = L – P (V – V ) ( m = 1)

2 1

V  V1 (b) W  PV  10 3  0.25  250 J


 P2  P1  1   10 5  (2)1.3 ( V2  ) 8.
 V2  2
9. (d) W  PV  1.01  10 5 (3.34  2  10 3 )
49. (b) In adiabatic process U = – W. In compression W is
negative, so U is positive i.e. internal energy increases.  337  10 3 J  340 KJ
50. (a) According to the first law of thermodynamics T2 V2
Q  U  W 10. (c)   2  T2  2  T1  2  300  600 K  327 o C
T1 V1
In adiabatic process Q  0 , hence U  W
 
11. (c) V  T at constant pressure
P V   V 
51. (b) PV  constant  2   1    1   4 

V1 T1 VT 300  280
P1  V2   V1 / 4     V2  1 2   280 ml.
V2 T2 T1 300
 P2  4  P
12. (a) In thermodynamic process, work done is equal to the area
As  is always greater than one so 4   4  P2  4 P covered by the PV curve with volume axis.
 Hence, according to graph shown
P2  V1 
3/2
4  8
52. (d) P1V1  P2 V2       Wadiabatic  Wisothermal  Wisobaric
P1  V2  1  1
53. (b) Change in internal energy of the gas P
Isobaric
U   W
R
T2  T1   8.3 [308  300]  166 J
 1 (1 .4  1) Isothermal
Adiabatic

V1 V2 V
684 Thermodynamics

T2 (273  69)
11. (c)   1  1  0.5
T1 (273  411)
 Work done    Q  0.5  1000  500 J
13. (b) (Similar to previous question)
T2 W Q  Q2
14. (a) 12. (b)   1    1
T1 Q1 Q1
15. (d) W  PV  2.4  10 4  1  10 5 =24J
where Q1  heat absorbed, Q2  heat rejected
16. (b) At constant pressure
T /3 W 2 W Q  Q2
W  PV  RT  1  8.31  100  831  814 J  1     1
T Q1 3 Q1 Q1
17. (a) V  0  PV  0  W  0
2 Q Q 1 Q Q
18. (d) Entropy of a reversible process does not change.  1 2  2   Q2  1 
3 Q1 Q1 3 3 3
19. (c) W  PV  0 (As V  0 ) T2 25 300 1 300
13. (c)   1   1  1
20. (d) T1 100 T1 4 T1

21. (d) At constant volume P  T 


P1 T1

P
 1 
300 3
 T1  400 K  127C
P2 T2 P2 400 4
T2 70 T
14. (a)   1    1  2  T2  300 K
22. (d) In isothermal process Q  0. T1 100 1000
23. (a) For isochoric process V  0  W = 0 T2 T1  T2 (473  273) 200
15. (a)   1    1  
From FLOT Q  U  W  Q  U T1 T1 473 473
273  73 200
and 2  
Heat Engine, Refrigerator and 273 273
Second Law of Thermodynamics 1 273
So required ratio   0 .577
2 473
T1  T2 W  T1 
1. (d)     Q   W
 T2 (273  123) 150 1
T1 Q  T1  T2  16. (a)   1  1 1   50%
T1 (273  27) 300 2
600
  800 =1600 J T2 300 2
(600  300) 17. (b)   1  1 
T1 500 5
2. (c) Coefficient of performance
18. (d)
T2 273 273 500 3
K   9 19. (b) In first case, (1 )  1  
T1  T2 303  273 30 800 8
3. (b) In a refrigerator, the heat dissipated in the atmosphere is more 600
than that taken from the cooling chamber, therefore the room and in second case, ( 2 )  1 
x
is heated if the door of a refrigerator is kept open.
3 600
4. (c) Internal energy is a state function. Since 1  2 , therefore 1
8 x
5. (b)
600 3 5 600  8
or  1   or x   960 K
dQ
6. (d) For a reversible process  T
0 x
T
8 8
300 1
5

20. (a) max  1  2  1    25%


7. (b) For cyclic forces U  0 So, Q  W T1 400 4
So 26% efficiency is impossible
T2 400 1 W 1 W
8. (d)   1  1      T (273  0) 200
T1 500 5 Q 5 Q 21. (b) In first case 1  1  2  1  
T1 (273  200) 473
Q 6
 W   10 4  1.2  10 4 J (273  200) 200
5 5 In second case 2  1  
(273  0) 273
9. (b)   1 
T2

30
1
350 1 1
   1 : 1 .73
T1 100 T1  2  473 
 
350 50 70 7  273 
 1    T1  500 K  227C
T1 100 100 10 T 1 500 500 1
22. (b)   1  2   1    …..(i)
T1 2 T1 T1 2
T
10. (b)   1  2 for 100% efficiency  = 1 which gives T = 0 K. 60 T' T ' 2
T1 1 2  2 
2

…..(ii)
100 T1 T1 5
Thermodynamics 685

500 5 3. (b,c) There is a decrease in volume during melting on an ice slab at


Dividing equation (i) by (ii),   T2 '  400 K 273K. Therefore, negative work is done by ice-water system on
T2 ' 4 the atmosphere or positive work is done on the ice-water
23. (c) system by the atmosphere. Hence option (b) is correct.
24. (a) Secondly heat is absorbed during melting (i.e. Q is positive)
T2 W  T   (273  27)  and as we have seen, work done by ice-water system is negative
25. (b)   1    W   1  1  Q  1   ( W is negative). Therefore, from first law of
T1 Q  T2   (273  627) 
thermodynamics U  Q  W.
 W  1 
300 
  3  10 6  2  10  4.2 J  8.4  10 J
6 6
Change in internal energy of ice-water system, U will be
 900 
positive or internal energy will increase.
26. (a)
4. (c) Process is isothermal. There fore, T = constant,
T T
27. (d)   1  2 ; for  to be max. ratio 2 should be min.  1
T1 T1  P   volume is increasing, therefore pressure will
28. (a)  V 
decreases.
T  T2 In chamber A :
29. (b) In first case 1  1
T1  RT  A RT  A RT
P  Pi  Pf  A   …..(i)
2T1  2T2 T  T2 V 2V 2V
In second case  2   1 
2T1 T1 In chamber B :
30. (c) Coefficient of performance  RT  B RT  B RT
1.5 P  Pi  Pf  B   …..(ii)
T2 (273  13) 260 V 2V 2V
K 5   
T1  T2 T1  (273  13) T1  260 from equations (i) and (ii) A 
1

2
 B 1.5 3
 5 T1  1300  260  5 T1  1560
mA / M 2
 T1  312K  39C    3m A  2m B .
mB / M 3
T2
31. (a) Coefficient of performance K   1
5
1
2
T1  T2  1  1 T V  L A3 L 3
5. (d) T1V1  T2 V2  1   2    2    2 
(273  23) 250 250 T2  V1   L1 A   L1 
   5
(273  27)  (273  23) 300  250 20 V
T1  T2 (273  727)  (273  227) 1000  500 1 6. (d) Using Boyle’s law, we have  constant
32. (a)      T
T1 273  727 1000 2 l l
5 5
T1  T2 W Q(T1  T2 )  2  2
33. (c)     W
T1 Q T1 373 273
As the piston moves 5 cm, the length of one side will be
6  10 4 (227  273)  (273  127)
 l  l 
(227  273)   5  and other side   5  . On solving this equation,
2  2 
6  10 4  100 we get l = 64.6 cm.
  1.2  10 4 cal
500
34. (d) Slow isothermal expansion or compression of an ideal gas is
reversible process, while the other given process are irreversible
in nature.

Critical Thinking Questions


1. (d) Fraction of supplied energy which in creases the internal
energy is given by
U (Q)V C V T 1
f   
(Q)P (Q)P C P T 
7 5
For diatomic gas    f
5 7
2. (c) Q  U  W
P(V2 / V1 )
 U  Q  W  540 
J
1.013  10 5  [(1671  1)  10 6 ]
 540 
4 .2
 540  39.7  500 calories
686 Thermodynamics

 (Q)V   3 5   5 
7. (c) Q  U  W  W  (Q)P  U  (Q)P 1     4  RT  2  RT    4  RT  = RT
 (Q)P   2 2   2 
Note : (a) 2 moles of diatomic gas becomes 4 moles of a
 C   3 2 monoatomic gas when gas dissociated into atoms.
 (Q)P 1  V   Q  1    Q
 CP   5 5 (b) Internal energy of  moles of an ideal gas of degrees of
f
5 freedom F is given by U  RT
 (Q)P  Q and   for monatomic gas 2
3
F = 3 for a monoatomic gas and 5 for diatomic gas.
8. (d) Oxygen is diatomic gas, hence its energy of two moles
5 14. (c) PV  K or P V  1dV  dP. V   0
 2  RT  5 RT
2 dP dV dP  dV 
or   or  100     100 
Argon is a monoatomic gas, hence its internal energy of 4 P V P  V 
3
moles  4  RT  6 RT  1.4  5  7%
2 15. (a) TV  1 = constant
Total Internal energy  (6  5)RT  11RT  1  1
T1  V2  1 1
    or   
9. (c) From graph it is clear that P3  P1 . T2  V1  2 2
Since area under adiabatic process (BCED) is greater than that 1 3
of isothermal process (ABDE). Therefore net work done    1  or    PV 3 / 2 = constant
2 2
W  Wi  (WA )  WA  Wi  W  0 T  T2 WA T  T3 WB
16. (c)  A  1   B  2 
C T1 Q1 T2 Q2
P3
Q1 T T  T3 T1
P1   1  2   WA  WB
A Q 2 T2 T1  T2 T2
P2 B T1  T3 800  300
 T2    550 K
2 2
E(V1) D(V2) V 17. (b) For monoatomic gas
10. (a) According to given Vander Waal’s equation
C 5
  P  we know Q  nC P T
nRT n 2 CV 3
P  2
V  n V U C V 3
and U  nCV T   
V2 V2 dV V2 dV Q C P 5
Work done, W  V1
PdV  nRT V1 V  n
 n 2 
V1 V2 i.e. fraction of heat energy to increase the internal energy be
3/5.
V
1  2
 nRT loge (V  n  ) VV12  n 2   18. (a) Q  U  W 
W
1 
U
1 
nC V dT
 V  V1 Q Q nC P dT

V2  n  V  V2  W C 3 2
 nRT loge  n 2  1    1  V  1    0.4
V1  n   Q CP 5 5
 V1 V2 
m N
11. (b) Volume of the gas is constant V = constant  P  T 19. (b) U  CV T  CV T  CV T
M NA
i.e., pressure will be doubled if temperature is doubled
 P  2P0 56  10 3 5
F
 (U)N   R  300
14 2
Now let F be the tension in the wire. Then
equilibrium of any one piston gives PA P 0A 6  10 26 3
and (U)A   R  900  (U)N  (U)A
F  (P  P0 )A  (2 P0  P0 )A  P0 A 6  10 23 2
20. (c) A is compressed isothermally, hence
5  2(dU) V
12. (c) dU  C V dT   R dT or dT  V  P2  P2  2 P1
P1…..(i)
 2  5R 2
From first law of thermodynamics and B is compressed adiabatically, hence

Q 3Q V
dU  dQ  dW  Q   . Now molar heat capacity P1V   P2    P2  (2) P1
4 4 2
C
dQ

Q

5 RQ

10
R. Since   1 , hence P2 '  P2 or P2  P2
dT 2(dU)  3Q  3
2  21. (b) In isothermal process P1 V1  P2 V2
5R  4 
P
13. (b) Q  U  U f  Ui = [internal energy of 4 moles of a or PV  P2  4 V  P2 
4
monoatomic gas + internal energy of 2 moles of a diatomic gas] In adiabatic process
– [internal energy of 4 moles of a diatomic gas]
Thermodynamics 687

  P V0 V
P2 V2  P3 V3   (4 V )1.5  P2 V 1.5  P3  2 P VL   Ax and VR  0  Ax
4 2 2
22. (d) Volume of the ideal gas is constant so W  PV  0 using As it is given that the container walls and the piston are
adiabatic in left side and the gas undergoes adiabatic expansion
FLOT Q  U  U  i2 Rt  1 2  100  5  60
and on the right side the gas undergoes adiabatic compressive.
 30  10 3  30 KJ Thus we have for initial and final state of gas on left side
 
 T  W 1 V  V 
23. (d) Initially   1  2    ...(i) P1  0   Pf  0  Ax  .....(i)
 T1  Q 6  
2  2 
 T '  (T  62)  T 62 Similarly for gas in right side, we have
Finally '  1  2    1  2  1 2 
  
 T1   T1  T1 T1 V  V 
P2  0   Pf  0  Ax  .....(ii)
62  2   2 
  ....(ii)
T1 From eq. (i) and (ii)

It is given that '  2. Hence solving equation (i) and (ii)  V0 
 T1  372 K  99C and T2  310 K  37C P1

 2
 Ax 
 V P1 /   P21 /   
1 g 2kcal

P2  V0


 Ax 

 Ax  0 11 / 
2 P1  P21 /   
24. (b) Input energy    2kcal / sec .
sec g  2 

Output energy  10 KW  10 K J / S 
10
kcal / sec . V 
4.2 P1  0 
Now from equation (i) Pf   2 
output energy 10 
   1, it is impossible.  V0 
input energy 4 .2  2 2  Ax 
 
25. (b) Gain of entropy of ice
30. (d) In both cylinders A and B the gases are diatomic ( = 1.4).
Q mL 80  100 8  10 3 Piston A is free to move i.e. it is isobaric process. Piston B is
S1     cal / K
T T (0  273) 273 fixed i.e. it is isochoric process. If same amount of heat Q is
Q mL given to both then
Loss of entropy of water  S 2   
T T (Q)isobaric  (Q)isochoric   C p (T )A   Cv (T )B
80  100 8  10 3
Cp
  cal / K  (T )B  (T )A   (T )A  1.4  30  42 K.
(273  50) 323 Cv
Total change of entropy
8  10 3 8  10 3 Graphical Questions
S1  S 2    4.5 cal / K
273 323
26. (b) PV 2  constant represents adiabatic equation. So during the 1. (c) As internal energy is a point function therefore change in
expansion of ideal gas internal energy of gas decreases and internal energy does not depends upon the path followed i.e.
temperature falls. UI  UII
T  T2 T  (273  7) 2. (b) Work done by the system = Area of shaded portion on P-V
27. (d) Initially   1  0.5  1
T1 T1 diagram
1 T1  280  (300  100)10 6  (200  10)  10 3  20 J
   T1  560 K
2 T1 3. (a) Work done = Area enclosed by triangle
T '  T2 T '  (273  7) 1 1
ABC  AC  BC   (3 V  V )  (3 P  P)  2 PV
Finally 1 '  1  0.7  1  T1 '  933 K
T1
 T 1
2 2
4. (c) Area enclosed between a and f is maximum. So work done in
 increase in temperature  933  560  373 K  380 K closed cycles follows a and f is maximum.
28. (c) P-V diagram of the gas is a straight line passing through origin.
5. (a) Initial and final states are same in all the process.
Hence P  V or PV 1  constant
Hence  U = 0; in each case.
Molar heat capacity in the process PV x  constant is
By FLOT; Q = W = Area enclosed by curve with volume axis.
R R
C  ; Here   1.4 (For diatomic gas)  (Area) < (Area) < (Area)  Q < Q < Q .
 1 1  x 1 2 1 1 2 3

6. (a) For an isothermal process PV = constant


R R
 C   C  3R 1  dV  1
1.4  1 1  1  PdV  VdP  0    
29. (c) As finally the piston is in equilibrium, both the gases must be V  dP  P
at same pressure Pf . It is given that displacement of piston be 1
So,    graph will be rectangular hyperbola.
in final state x and if A is the area of cross-section of the P
piston. Hence the final volumes of the left and right part finally
can be given by figure as 7. (a) By adjoining graph W AB  0 and

PT
1 1
PT
2 2
688 Thermodynamics

WBC  8  10 4 [5  2]  10 3  240 J 17. (a) Q1  T0 S 0 


1 3
T0 S 0  T0 S 0
2 2
 WAC  WAB  WBC  0  240  240 J T
Q2  T0 S 0 and Q3  0
Now, Q AC  Q AB  Q BC  600  200  800 J 2T0
Q Q
W Q  Q2 1

  1
3

From FLOT Q AC  U AC  WAC Q1 Q1


T0
 800  U AC  240  U AC  560 J . Q 2 1 Q
1 2 1 
2

8. (b) In adiabatic process, slope of PV-graph. Q1 3 3 S 0 2 S0 S

dP P 18. (a) Work done = Area of closed PV diagram


   |Slope|   = (2V  V )  (2 P  P)  PV
dV V
19. (c) From the given VT diagram,
From the given graph (Slope) > (Slope)   2   1
In process AB, V  T  Pressure is constant (As quantity of
2 1

therefore 1 should correspond to O ( = 1.4) and 2 should the gas remains same)
2

In process BC, V = Constant and in process CA,


correspond to He ( = 1.66)
T = constant
9. (c) As we know that slope of isothermal and adiabatic curves are  These processes are correctly represented on PV diagram by
always negative and slope of adiabatic curve is always greater graph (c).
than that of isothermal curve
20. (d) Q  U  W ; U does not depend upon path.
Hence in the given graph curve A and B represents adiabatic
and isothermal changes respectively.  WA  WB  Q A  QB
10. (d) Process CD is isochoric as volume is constant, Process DA is 6 P1  3 V1
21. (d) Work done = Area under curve  = 9 PV
isothermal as temperature constant and Process AB is isobaric 2
1 1

as pressure is constant.
1
11. (d) Heat given Q  20 cal  20  4.2  84 J . 22. (d) Work done   2 P1  2 V1  2 P1V1
2
Work done W = – 50 J [As process is anticlockwise] 23. (c) In a cyclic, U = 0
By first law of thermodynamics From FLOT,  Q = U + W = 0 + W = Area of closed curve
 U  Q  W  84  ( 50)  134 J 2
 20 
 Q = r    kPa  litre
(a) For cyclic process. Total work done  WAB  WBC  WCA
2

12.  2 
W = PV = 10(2 – 1) = 10J and W =0
AB BC  100   10 3  10 3 J  100 J
(as V = constant) 24. (c) The work done in cyclic process is equal to the area enclosed
by the PV diagram
From FLOT, Q = U + W
25. (d) In all given cases, process is cyclic and in cyclic process U = 0.
U = 0 (Process ABCA is cyclic) 26. (b) In cyclic process Q = Work done = Area inside the closed
curve.
 Q = W + W + W

AB BC CA

Treat the circle as an ellipse of area  (P2  P1 ) (V2  V1 )


 5 = 10 + 0 + W  W = – 5 J
CA CA 4
13. (b) The cyclic process 1 is clockwise where as process 2 is  
anticlockwise. Clockwise area represents positive work and  Q  {(150  50)  10 3 }  J
4 2
anticlockwise area represents negative work. Since negative area
(2) > positive area (1), hence net work done is negative. P0 V0
27. (d) W = – Area of triangle BCO  
14. (c) Process AB is isochoric,  WAB  P V  0
BCOB

2
V  P0 V0
Process BC is isothermal  WBC  RT2 . ln 2  W = + Area of triangle AOD  

AOD A

2
 V1 
28. (c) AD and BC represent adiabatic process (more slope)
Process CA is isobaric AB and DC represent isothermal process (less slope)
 WCA   PV   RT   R(T1  T2 )  R(T2  T1 ) 29. (c) Work done = Area of curve enclosed
(Negative sign is taken because of compression)  2V  2 P  4 PV
15. (a) AB is isobaric process, BC is isothermal process, CD is 30. (c) Work done = Area of PV graph (here trapezium)
isometric process and DA is isothermal process
1
These process are correctly represented by graph (a).  (1  10 5  5  10 5 )  (5  1)  12  10 5 J
2
16. (c) Work done by the gas (as cyclic process is clockwise)  W =
Area ABCD 31. (d) For path ab : (U)ab  7000 J
So from the first law of thermodynamics Q (net heat By using U  CV T
absorbed) = W = Area ABCD
5
As change in internal energy in cycle U = 0. 7000    R  700    0.48
2
For path ca :
Thermodynamics 689

(Q)ca  (U)ca  (W )ca ….(i) 1


40. (b) In isothermal process P  .
 (U)ab  (U)bc  (U)ca  0 V
Hence graph between P and V is a hyperbola.
 7000  0  (U)ca  0  (U)ca  7000 J ….(ii)
41. (d) Adiabatic curves are more stepper than isothermal curves.
Also (W )ca  P1 (V1  V2 )  R(T1  T2 )
42. (d) During process A to B, pressure and volume both are
 0.48  8.31  (300  1000)  2792.16 J ….(iii) decreasing. Therefore, temperature and hence, internal energy
of the gas will decrease (T  PV) or U A B  negative.
on solving equations (i), (ii) and (iii)
(Q)ca  7000  2792.16  9792.16 J  9800 J Further WA  B is also negative as the volume of the gas is
32. (b) Work done = Area enclosed by indicator diagram decreasing. Thus Q A  B is negative.
1 In process B to C pressure of the gas is constant while volume

 (3 V  V )(4 P  P)  3 PV is increasing. Hence temperature should increase or UB C =
2
positive. During C to A volume is constant while pressure is
33. (d) U , remains same for both path increasing. Therefore, temperature and hence, internal energy
For path iaf : U  Q  W  50  20  30 J . of the gas should increase or UC  A = positive. During
For path fi : U   30 J and W  13 J process CAB volume of the gas is decreasing. Hence, work done
by the gas is negative.
 Q   30  13   43 J .
43. (a) WAB  0 as V = constant
34. (a) Eint  0 , for a complete cycle and for given cycle work done
is negative, so from first law of thermodynamics Q will be  Q AB  U AB  50 J (Given)
negative i.e. Q  0 . U A  1500 J  UB  (1500  50)J  1550 J
35. (d) Work done = Area enclosed by the curve
WBC  UBC  40 J (Given)
1

(3 V  V ) (2 P  P)  PV
2  U BC  40 J  UC  (1550  40)J  1590 J
36. (d) W = Area bonded by the indicator diagram with V-axis)
44. (a) For adiabatic process T1 Vb 1 = Constant
1
 (PA  PB ) (VB  VA )  1
2 T2  Vb 
For bc curve T1 Vb 1  T2 Vc 1 or   …..(i)
37. (a) Heat given Q = 40 J and Work done W = 30 J T1  Vc 

 U = Q – W = 40 – 30 = 10 J.  1
38. (a) As the volume is continuously increasing and the work of T2  Va 
For ad curve T1 Va 1  T2 Vd 1 or   …..(ii)
expansion is always positive, so the work done by the system T1  Vd 

continuously increases.
39. (c) Processes A to B and C to D are parts of straight line graphs of Vb V
the form y = mx From equation (i) and (ii)  a
Vc Vd
R
Also P  T ( = 6)
V Assertion and Reason
 P  T. So volume remains constant for the graphs AB and
CD 1. (a) In a perfectly reversible system, there is no loss of energy.
P Losses can be minimised, friction can be reduced, the resistance
P2 B TB =800 K TC = 2200 K in L-C oscillating system can also be negligible. But one cannot
P2 C completely eliminate energy losses. This makes a perfectly
VB
VC reversible system, an ideal.
TA =600 K 2. (a) Adiabatic expansion produces cooling.
P1 TD =1200 K
A VA D 3. (a) In reversible process, there always occurs some loss of energy.
VD
This is because energy spent in working against the dissipative
force is not recovered back. Some irreversible process occur in
nature such as friction where extra work to cancel the effect of
T friction. Salt dissolves in water but a salt does not separate by
So no work is done during processes for A to B and C to D i.e., W = AB
itself into pure salt and pure water.
W = 0 and W = P (V – V) = R (T – T )
4. (a) When a bottle of cold carbonated drink is opened. A slight fog
CD BC 2 C B C B

= 6R (2200 – 800) = 6R  1400 J forms around the opening. This is because of adiabatic
Also W = P (V – V ) = R(T – T ) expansion of gas causes lowering of temperature and
DA 1 A D A B
condensation of water vapours.
= 6R (600 – 1200)= – 6R  600 J 5. (e) As isothermal processes are very slow and so the different
Hence work done in complete cycle isothermal curves have different slopes so they cannot intersect
W=W +W +W +W each other.
AB BC CD DA

6. (d) Adiabatic compression is a rapid action and both the internal


= 0 + 6R  1400 + 0 – 6R  600 energy and the temperature increases.
= 6R  900 = 6  8.3  800  40 kJ 7. (e) As there is no change in internal energy of the system during
an isothermal change. Hence, the energy taken by the gas is
690 Thermodynamics
utilised by doing work against external pressure. According to Hence, according to following graph.
FLOT Q  U  PV (Area) < (Area)  W < W
1 2 adi iso

Hence Q  U  PV Also in isothermal changes P


Therefore, reason is true and assertion is false. temperature remains same
8. (d) We can change the temperature of a body without giving (or but in adiabatic changes Isothermal
taking) heat to (or from) it. For example in an adiabatic temperature also changes.
2
compression temperature rises and in an adiabatic expansion 19. (c) First law of 1
temperature false, although no heat is given or taken from the thermodynamics is Adiabatic
system in the respective changes. restatement of the V
principal of conservation of energy as applied to heat energy.
Q
9. (a) c ; a gas may be heated by putting pressure, so it 20. (e) Zeroth law of thermodynamics explain the concept of
m . temperature. According to which there exist a scalar quantity
can have values for 0 to . called temperature which is property of all thermodynamic
C P and C V are it’s two principle specific heats, out of system.
infinite possible values. W T
21. (b) Efficiency of cannot cycle    1  2 , for Carnot
In adiabatic process C = 0 and in isothermal process C   . Q1 T1
10. (a) Heat is similar to work in that both represent ways of engine when T2 decrease  increases.
transferring energy. Neither heat nor work is an intrinsic 22. (a) Entropy is a measure of the disorder or randomness of the
property of a system, that is, we cannot say that a system system. Greater the randomness, greater the entropy.
contains a certain amount of heat or work.
11. (d) According to first law of thermodynamics,
Q  U  W  U  PV . If heat is supplies in such a
manner that volume does not change V  0 i.e., isochoric
process, then whole of the heat energy supplied to the system
will increase internal energy only. But, in any other process it is
not possible.
Also heat may absorbed or evolved when state of thermal
equilibrium changes.
12. (d) When the door of refrigerator is kept open, heat rejected by
the refrigerator to the room will be more than the heat taken
by the refrigerator from the room (by an amount equal to
work done by the compressor). Therefore, temperature of room
will increase and so it will be warmed gradually. As according
to 2 law of thermodynamics, heat cannot be transferred on its
nd

own, from a body at lower temperature to another at higher


temperature.
13. (a) Second law of thermodynamics can be explained with the help of
example of refrigerator, as we know that refrigerator, the
working substance extracts heat from colder body and rejects a
large amount of heat to a hotter body with the help of an
external agency i.e., the electric supply of the refrigerator. No
refrigerator can ever work without external supply of electric
energy to it.
14. (d) If an electric fan is switched on in a closed room, the air will be
heated because due to motion of the fan, the speed of air
molecules will increase. In fact, we feel cold due to evaporation
of our sweat.
15. (c) The internal energy of system depends only on its temperature.
In isothermal process temperature does not change, therefore,
internal energy of the system remains the same.
16. (c) In an adiabatic process, no exchange of heat is permissible i.e.,
Q  0 .
As, Q  U  W  0  U   W .
Also in adiabatic process, temperature of gas changes.
Q
17. (a) Change in entropy, S  . In an adiabatic change, heat
T
transfer Q  0 .  S  0 , or S  constant i.e., entropy
remains constant in an adiabatic process, or an adiabatic
process is an isoentropic process.
18. (b) As we know, in thermodynamic processes work done = Area
covered by P-V diagram with volume axis.
Thermodynamics 691

1. The P-V diagram of 2 gm of helium gas for a certain process 8. A tyre filled with air (27 o C, and 2 atm) bursts, then what is
A  B is shown in the figure. what is the heat given to the gas
temperature of air (  1.5) [RPMT 2002]
during the process A  B
P (a)  33 o C (b) 0 o C
(a) 4 Po Vo
2P0
B (c) 27 o C (d) 240 o C
(b) 6 Po Vo
9. A gas expands adiabatically at constant pressure such that its
1
(c) 4.5 Po Vo temperature T  , the value of CP / CV of gas is
P0 A V
(d) 2 Po Vo [RPMT 2002; MHCET 2004]
V0 2V0 V
2. A certain mass of gas at 273 K is expanded to 81 times its volume (a) 1.30 (b) 1.50
under adiabatic condition. If   1.25 for the gas, then its final (c) 1.67 (d) 2.00
temperature is [Pb. PET 1997] 10. P-V diagram of an ideal gas is as shown in figure. Work done by the
(a) – 235°C (b) – 182°C gas in process ABCD is
(c) – 91°C (d) 0°C (a) 4 P0 V0 P

3. In an adiabatic process 90J of work is done on the gas. The change C D


(b) 2 P0 V0 2P0
in internal energy of the gas is [CPMT 1996]
(a) – 90 J (c) 3 P0 V0 P0
B A
(b) +90 J
(d) P0 V0 V
(c) 0 J V0 2V0 3V 0
11. An engineer claims to have made an engine delivering 10 kW power
(d) Depends on initial temperature
with fuel consumption of 1 g s 1 . The calorific value of fuel is 2k
4. If a Carnot’s engine functions at source temperature 127°C and at
sink temperature 87°C, what is its efficiency cal/g. His claim [J & K CET 2000]

[DCE 1997] (a) Is non-valid (b) Is valid


(a) 10% (b) 25% (c) Depends on engine (d) Depends on load
(c) 40% (d) 50% 12. An ideal gas heat engine operates in a Carnot cycle between 27° C
5. In the case of diatomic gas, the heat given at constant pressure is and 127°C. It absorbs 6 kcal at the higher temperature. The amount
that part of energy which is used for the expansion of gas, is of heat (in kcal) converted into work is equal to
2 3 (a) 3.5 (b) 1.6
(a) (b)
5 7 (c) 1.2 (d) 4.8
2 5 13. A gas expands with temperature according to the relation
(c) (d)
7 7 V  kT 2 / 3 . What is the work done when the temperature changes
6. An ideal monoatomic gas is taken round the cycle ABCDA shown in by 30 o C
the PV diagram in the given fig. The work done during the cycle is [UPSEAT 1998]
(a) 10 R (b) 20 R
1 (c) 30 R (d) 40 R
(a) PV
2 P
14. An ideal gas (  1.5) is expanded adiabatically. How many times
A B
(b) 2 PV (P,2V) (2P,2V) has the gas to be expanded to reduce the root mean square velocity
of molecules 2.0 times
(c) PV
(a) 4 times (b) 16 times
(d) Zero
(PV) (P,2V)
(c) 8 times (d) 2 times
7. A gas is compressed adiabatically till D its temperature
C is doubled. The
V 15. Three samples of the same gas A, B and C(  3 / 2) have initially
ratio of its final volume to initial volume will be [BHU 1997]
equal volume. Now the volume of each sample is doubled. The
(a) 1 / 2 (b) More than 1 / 2 process is adiabatic for A isobaric for B and isothermal for C. If the
final pressures are equal for all three samples, the ratio of their
(c) Less than 1 / 2 (d) Between 1 and 2 initial pressures are
692 Thermodynamics
Finally the gas is compressed at constant volume to its original
(a) 2 2 : 2 :1 (b) 2 2 :1: 2
pressure PA . The correct P-V and P-T diagrams indicating the
(c) 2 :1: 2 (d) 2 :1: 2 process are
16. Volume versus temperature graph of two moles of helium gas is as P P
shown in figure. The ratio of heat absorbed and the work done by A C
the gas in process 1-2 is (a) PA (b) PA
(a) 3 V

5 2
(b) C A B
2 PA/2 B PA/2

5
(c) VA 2VA V VA 2VA V
3 1 P
P
7 A B PA A
(d) T (c) PA (d)
2
17. In the P-V diagram shown in figure ABC is a semicircle. The work
done in the process ABC is B
PA/2 C PA/2 C
(a) Zero P(atm)
3
 TA/2 TA TA/2 TA T
(b) atm  lt T
2 22. A cylinder of mass 1kg is given heat of 20000 J at atmospheric
pressure. If initially temperature of cylinder is 20°C, then work done

(c)  atm  lt 1 by the cylinder will be (Given that Specific heat of cylinder = 400 J
2
kg , Coefficient of volume expansion = 9  10 °C , Atmospheric
–1 –5 –1

(d) 4 atm-lt 1 2 V(litre) pressure = 10 N/m and density of cylinder 9000 kg/m )
5 2 3

18. Heat is supplied to a diatomic gas at constant pressure. The ratio of


(a) 0.02 J (b) 0.05 J
Q : U : W is
(c) 0.08 J (d) 0.1 J
(a) 5 : 3 : 2 (b) 5 : 2 : 3
23. In a thermodynamic process pressure of a fixed mass of a gas is
(c) 7 : 5 : 2 (d) 7 : 2 : 5 changed in such a manner that the gas releases 30 joules of heat
and 10 joules of work was done on the gas. If the initial internal
19. A gas undergoes a change of state during which 100 J of heat is energy of the gas was 30 joules, then the final internal energy will
supplied to it and it does 20 J of work. The system is brought back be [CPMT 1986]
to its original state through a process during which 20 J of heat is
released by the gas. The work done by the gas in the second process (a) 2 J (b) – 18 J
is (c) 10 J (d) 58 J
(a) 60 J (b) 40 J
24. In an adiabatic change, the pressure P and temperature T of a
(c) 80 J (d) 20 J
monoatomic gas are related by the relation P  T C , where c
20. N moles of an ideal diatomic gas are in a cylinder at temperature T. equals [CBSE PMT 1994;
suppose on supplying heat to the gas, its temperature remain BHU 1997; AIIMS 2001; MH CET 2000]
constant but n moles get dissociated into atoms. Heat supplied to
the gas is (a) 5 / 3 (b) 2 / 5

1 (c) 3 / 5 (d) 5 / 2
(a) Zero (b) nRT 25. The internal energy of an ideal gas increases during an isothermal
2
process when the gas is [SCRA 1998]
3 3 (a) Expanded by adding more molecules to it
(c) nRT (d) ( N  n)RT
2 2 (b) Expanded by adding more heat to it
 7  (c) Expanded against zero pressure
21. Three moles of an ideal gas  C P  R  at pressure PA and
 2  (d) Compressed by doing work on it
temperature T A is isothermally expanded to twice its initial volume.
It is then compressed at constant pressure to its original volume.

(SET -14)

1. (b) Change in internal energy from A  B is f f


U  RT  (Pf V f  Pi Vi )
2 2
693

3 9 T2
 (2 P0  2 V0  P0  V0 )  P0 V0 12. (c) Efficiency of a carnot engine is given by   1 
2 2 T1
Work done in process A  B is equal to the Area covered by W T W (273  127)
the graph with volume axis i.e., or 1 2  1  W  1.2 k cal
Q T1 6 (273  227)
1 3
W AB  (P0  2 P0 )  (2V0  V0 )  P0 V0
 PdV  
RT
2 2 13. (b) W  dV
V
9 3
Hence, Q  U  W  P0 V0  P0 V0  6 P0 V0 2
2 2 Since V  kT 2 / 3  dV  KT 1 / 3 dT
3
2. (b) For adiabatic process TV  1 = constant
dV 2 dT
T V 
 1
V 
 1 Eliminating K , we find 
 2   1 
  T2   1 
  T1 V 3 T
T1  V2   V2  T2 2 RT 2 2
 1 
1 .25 1
 1 
0 .25
Hence W  
T1 3 T
dT  R(T2  T1 )  R(30)  20 R
3 3
 T2     273     273
 81   81  3 RT
14. (b) v rms   v rms  T
273 M
  91K  –182°C
3 v rms is to reduce two times i.e. temperature of the gas will
3. (b) For adiabatic process Q  0 T 1
have to reduce four times or 
From Q  U  W  0  U  90  U  90 J T 4
T1  T2 (127  273)  (87  273) During adiabatic process TV  1  T  V   1
4. (d)   
T1 (127  273) 1
1
400  360 V   T   1
  0.1  10%     (4 ) 1.5 1  (4 )2  16  V   16 V
400 V  T 
5. (c) W  energy used for expansion  PdV  RdT 15. (b) Let the initial pressure of the three samples be PA , PB and
Q  heat supplied to diatomic gas at constant P PC , then PA (V )3 / 2  (2V )3 / 2 P , PB  P and
7 7 W RdT 2
 C p dT  RdT ( C p  R)    PC (V )  P(2V )
2 2 Q 7 7
RdT
2  PA : PB : PC  (2)3 / 2 :1 : 2  2 2 :1 : 2
6. (c)
1
16. (b) V-T graph is a straight line passing through origin. Hence,
 1  1
T2  V1  V  1 V 1   1 1 V  T or P  constant
7. (c)    2   2    2   
T1  V2 
  V1

 2 V1  2  2  Q  nC P T and U  nC V T
V1 Also W  Q  U   (C P  C V ) T
 V2 
2
Q nC P T CP 1
 1 1 .5 1 1    
T2  P2   1  1 .5 13 W n (C P  C V ) T C P  CV C
8. (a)   
T2
     
1 1 V
T1  P1  (273  27)  2  2 2 .5 CP

(273  27) CV 3 Q 1 5
 T2 
T1
  238 K   34.8 C  for helium gas. Hence  
1.25 1.25 CP 5 W 1  3 / 5 2

9. (b) TV  1  constant  T  V 1 17. (b) W AB is negative (volume is decreasing) and


1
 WBC is positive (volume is increasing) and
According to question T  V 2
1 3 since, WBC  WAB
Hence 1        1 .5
2 2  net work done is positive and area between semicircle
10. (c) WAB   P0 V0 , WBC  0 and WCD  4 P0 V0 
which is equal to atm  lt.
2
 WABCD   P0 V0  0  4 P0 V0  3 P0 V0
7  7 
10000 18. (c) Q  C P T  RT CP  R 
11. (a) Power  10 KW  10000 J /s   2.38 k cal / gm 2  2 
4 .2
But the calorific value of fuel is only 2 k cal/gm. Hence claim is 5  5 
U  C V T  RT  CV  R 
invalid. 2  2 
and W  Q  U  RT
694 Thermodynamics

 Q : U : W  7 : 5 : 2  1 
 (10 5 N / m 2 )  m 3  (9  10 5 / C) (50C)  0 .05 J
 9  10
3
19. (a) In a cyclic process U  0  Q  W 

 (100  20)  20  W2  W2  60 J 23. (c) Q  U  W  (U f  Ui )  W

20. (b) Since the gas is enclosed in a vessel, therefore, during heating  30  (U f  30)  10  U f  10 J
process, volume of the gas remains constant. Hence, no work is

done by the gas. It means heat supplied to the gas is used to
increase its internal energy only. 24. (d) T  P 1  = constant  P  T  1
Comparing above equation with given equation
5 
Initial internal energy of the gas is U1  N  R  T  5/3 5
2  P  TC  C   
 1 5 / 3 1 2
Since n moles get dissociated into atoms, therefore, after
heating, vessel contains (N  n) moles of diatomic gas and 25. (a) Internal energy of an ideal gas is given by
2n moles of a mono-atomic gas. Hence the internal energy for f f N 
the gas, after heating, will be equal to U RT    RT  U  NT.

2 2  NA 
5  3  5 1 In isothermal process T = constant  U  N.
U 2  ( N  n) R  T  2n  R  T  NRT  nRT
 2   2  2 2 i.e. internal energy increases by increasing number of molecules
Hence, the heat supplied = increase in internal energy (N).
1
 (U 2  U1 )  nRT
2

***

21. (a) Let the process start from initial pressure PA , volume V A
and temperature T A .
P 
A(PA, VA, TA) B  A , 2 VA , TA 
 2 

P T 
C  A , VA , A 
 2 2 
(i) Isothermal expansion (PV  constant) at temperature
T A to twice the initial volume V A
PA
(ii) Compression at constant pressure to original volume
2
VA (i.e. V  T)
(iii) Isochoric process (at volume V A ) to initial condition
(i.e. P  T )
20000 J
22. (b) Q  mcT  T   50C
1kg  (400 J / kg C)
 T = 70°C
Final

Hence W  PatmV  PatmV0 T

You might also like